Hall Pain Questions Flashcards Preview

Pain Medicine Board Review MJ > Hall Pain Questions > Flashcards

Flashcards in Hall Pain Questions Deck (113)
Loading flashcards...
1
Q
788. Tachyphylaxis to local anesthetics is most closely related to which of the following?
A. Speed of injection
B. Dosing interval
C. Temperature of local anesthetic
D. Volume of local anesthetic
E. pH of solution
A
  1. (B) Tachyphylaxis is a well-known phenomenon associated with repeated injections of local anesthetics leading to
    decreased effectiveness. Interestingly, the dosing interval seems most important in the development of tachyphylaxis.
    If the dosing interval is short (and no pain between injections) tachyphylaxis does not develop. However,
    with longer dosing intervals (and pain between injections) tachyphylaxis develops (Barash: Clinical Anesthesia,
    ed 5, pp 458-459).
2
Q
789. Which of the following techniques is LEAST effective in a treatment of pruritus from administration of neuraxial opiates?
A. Nalbuphine 5 mg IV
B. Dexmedetomidine 30 μg IV
C. Diphenhydramine 50 mg IV
D. Hydroxyzine 20 mg IM
E. Propofol 10 mg IV
A
  1. (B) The treatment of pruritus, the most common side effect of neuraxial opiates, is primarily with opioid antagonists,
    mixed opioid agonist-antagonist, and antihistamine drugs (by their sedating effects). Nalbuphine is a
    mixed opioid agonist-antagonist; diphenhydramine and hydroxyzine have antihistamine properties. Propofol at
    very low doses (e.g., 10 mg) has been useful to treat pruritus not only induced by neuraxial opiates but also the
    pruritus associated with cholestatic liver disease. Propofol does not affect analgesia, whereas opioid antagonists
    and mixed agonist-antagonist may reverse some or all of the analgesia, depending upon dose. Dexmedetomidine
    is a highly selective α2-receptor agonist that has a faster onset and shorter duration of action compared with
    clonidine. Dexmedetomidine has analgesic properties, can potentiate neuraxial analgesia when injected spinally,
    and can perhaps decrease the incidence of pruritus by reducing the narcotic dose is used. It does not treat pruritus
    (Miller: Anesthesia, ed 6, p 2740; Stoelting: Pharmacology and Physiology in Anesthetic Practice, ed 4, pp 91,
    158, 434).
3
Q
  1. The maximum dose of lidocaine containing 1:200,000 epinephrine that can be administered to a 70-kg patient for
    regional anesthesia (other than spinal anesthesia) is
    A. 50 mg
    B. 100 mg
    C. 200 mg
    D. 500 mg
    E. 1000 mg
A
  1. (D) The maximum dose of local anesthetics containing 1:200,000 epinephrine that can be used for major nerve
    blocks is lidocaine, 500 mg; mepivacaine, 500 mg; prilocaine, 600 mg; bupivacaine, 225 mg; etidocaine,
    400 mg; and tetracaine, 200 mg (Miller: Anesthesia, ed 6, pp 584-589).
4
Q
791. Which of the following concentrations of epinephrine corresponds to a 1:200,000 mixture?
A. 0.5 μg/mL
B. 5 μg/mL
C. 50 μg/mL
D. 0.5 mg/mL
E. None of the above
A
  1. (B) 1:200,000 means 1 g = 1000 mg = 1,000,000 μg per 200,000 mL
    1,000,000 μg/200,000 mL = 5 μg/mL
5
Q
  1. An anesthesia pain service consult is sought for a 78-year-old patient with a complaint of pain in the distribution of
    the trigeminal nerve. The patient has no other medical problems except a history of congestive heart failure for which
    he takes digoxin and thiazide. In addition to his chief complaint, the patient over the last 72 hours has complained of
    dysesthesia in the feet, difficulty with vision, and emesis times three. The most appropriate step at this time would be
    A. Trigeminal nerve block with bupivacaine
    B. Obtain neurologic workup for multiple sclerosis
    C. Administration of fentanyl and ondansetron
    D. Initiate therapy with carbamazepine
    E. Obtain a digoxin level
A
  1. (E) The early signs of digitalis toxicity include loss of appetite and nausea and vomiting. In some patients there may
    be pain that is similar to trigeminal neuralgia. Pain or discomfort in the feet and pain and discomfort in the
    extremities may be a feature of digitalis toxicity. Transient visual disturbances (e.g., amblyopia, scotomata) have
    been reported in patients with digitalis toxicity. In this patient, it would be prudent to obtain a digoxin level as
    an early part of the workup for these complaints. He may also have true trigeminal neuralgia, and workup for
    this condition can be undertaken after digitalis toxicity has been ruled out (Stoelting: Pharmacology and Physiology
    in Anesthetic Practice, ed 4, pp 314-315).
6
Q
793. Which of the following is the earliest sign of lidocaine toxicity?
A. Shivering
B. Nystagmus
C. Lightheadedness and dizziness
D. Tonic-clonic seizures
E. Nausea and vomiting
A
  1. (C) Toxic reactions to local anesthetics are usually due to intravascular or intrathecal injection or to an excessive dosage.
    The initial symptoms of local anesthetic toxicity are lightheadedness and dizziness. Patients also may note
    perioral numbness and tinnitus. Progressive central nervous system (CNS) excitatory effects include visual and
    auditory disturbances, shivering, muscular twitching, and ultimately, generalized tonic-clonic seizures. CNS
    depression can ensue, leading to respiratory depression or arrest (Miller: Anesthesia, ed 6, pp 592-598).
7
Q
794. An analgesic effect similar to the epidural administration of 10 mg of morphine could be achieved by which dose of
intrathecal morphine?
A. 0.1 mg
B. 1 mg
C. 5 mg
D. 10 mg
E. There is no correlation
A
  1. (B) The site of action of spinally administered opiates is the substantia gelatinosa of the spinal cord. Epidural
    administration is complicated by factors related to dural penetration, absorption in fat, and systemic uptake;
    therefore, the quantity of intrathecally administered opioid required to achieve effective analgesia is typically
    much smaller. The ratio of epidural to intrathecal dose of morphine is approximately 10:1. Morphine is typically
    given in doses of 3 to 10 mg in the lumbar epidural space. Intrathecal morphine dosage is 0.2 to 1.0 mg.
    Onset time for epidural administration is 30 to 60 minutes with a peak effect in 90 to 120 minutes. Onset time
    for intrathecal administration is shorter than epidural administration. Duration of 12 to 24 hours of analgesic
    effect can be expected by either route (Barash: Clinical Anesthesia, ed 5, pp 1422-1423).
8
Q
795. Which local anesthetic undergoes the LEAST hepatic clearance
A. Chloroprocaine
B. Bupivacaine
C. Etidocaine
D. Prilocaine
E. Lidocaine
A
  1. (A) Commonly injected local anesthetics are divided chemically into two groups: the aminoesters (esters) and the
    aminoamides (amides). The esters include procaine, chloroprocaine and tetracaine (all have one letter i in the
    name). The amides are lidocaine, mepivacaine, prilocaine, bupivacaine, levobupivacaine, etidocaine and ropivacaine
    (all have two i’s in the name). The esters undergo plasma clearance by cholinesterases and have relatively
    short half-lives, whereas the amides undergo hepatic clearance and have longer half-lives (Barash: Clinical Anesthesia,
    ed 5, p 462).
9
Q
  1. Which of the following is the most important disadvantage of interscalene brachial plexus block compared with other
    approaches?
    A. Not suitable for operations on the shoulder
    B. Large volumes of local anesthetics required
    C. Frequent sparing of the ulnar nerve
    D. Frequent sparing of the musculocutaneous nerve
    E. High incidence of pneumothorax
A
  1. (C) The major disadvantage of the interscalene block for hand and forearm surgery is that blockade of the inferior
    trunk (C8-T1) is often incomplete. Supplementation of the ulnar nerve often is required. The risk of pneumothorax
    is quite low, but blockade of the ipsilateral phrenic nerve occurs in up to 100% of blocks. This can cause
    respiratory compromise in patients with significant lung disease (Miller: Anesthesia, ed 6, pp 1686-1689).
10
Q
  1. A 68-year-old woman is to undergo foot surgery under spinal anesthesia. Which of the following statements concerning
    the immediate physiologic response to the surgical incision is true?
    A. The cardiovascular response to stress will be blocked, but the adrenergic response will not
    B. The adrenergic response to stress will be blocked, but the cardiovascular response will not
    C. Both the adrenergic and cardiovascular responses will be blocked
    D. Neither the adrenergic or cardiovascular response will be blocked
    E. The cardiovascular response will be blocked but the adrenergic response will be augmented
A
  1. (C) Surgical trauma includes a wide variety of physiologic responses. General anesthesia has no or only a slight
    inhibitory effect on endocrine and metabolic responses to surgery. Regional anesthesia inhibits the nociceptive
    signal from reaching the CNS and, therefore, has a significant inhibitory effect on the stress response, including
    adrenergic, cardiovascular, metabolic, immunologic, and pituitary. This effect is most pronounced with procedures
    on the lower part of the body and less with major abdominal and thoracic procedures. The variable effect
    is probably due to unblocked afferents, i.e., vagal, phrenic, or sympathetic (Barash: Clinical Anesthesia, ed 5,
    p 1150).
11
Q
798. The “snap” felt just before entering the epidural space represents passage through which ligament?
A. Anterior longitudinal ligaments
B. Posterior longitudinal ligaments
C. Ligamentum flavum
D. Supraspinous ligament
E. Interspinous ligament
A
  1. (C) The structures that are traversed by a needle placed in the midline prior to the epidural space are as follows: skin,
    subcutaneous tissue, supraspinous ligament, interspinous ligament, and ligamentum flavum. The ligamentum
    flavum is tough and dense and a change in the resistance to advancing the needle is often perceived and to many
    feels like a “snap.” The anterior and posterior longitudinal ligaments bind the vertebral bodies together. See also
    explanation and diagram in question 870 (Barash: Clinical Anesthesia, ed 5, pp 698-699).
12
Q
799. The common element thought to be present in every case of cauda equina syndrome after continuous spinal anesthesia
is
A. Use of microcatheter
B. Maldistribution of local anesthetic
C. Administration of lidocaine
D. Addition of epinephrine
E. Hyperbaricity
A
  1. (B) The symptoms of cauda equina syndrome include low back pain, bilateral lower extremity weakness, saddle
    anesthesia and loss of bowel and bladder control. Pooling of local anesthetics in dependent areas of the spine
    within the subarachnoid space has been identified as the causative factor in cases of cauda equina syndrome.
    Microlumen catheters may enhance the nonuniform distribution of solutions within the intrathecal space, but
    cauda equina syndrome has been associated with the use of larger catheters, 5% lidocaine with dextrose, and 2%
    lidocaine, as well as 0.5% tetracaine (Barash: Clinical Anesthesia, ed 5, p 712; Stoelting: Basics of Anesthesia, ed 5,
    pp 132, 631-632).
13
Q
  1. A sciatic nerve block is performed in a healthy 26-year-old male patient for bunion surgery. Fifteen mL of 1.5% mepivacaine
    is slowly injected after the landmarks are identified and a paresthesia is elicited in the great toe. In what order
    would the following nerve fibers be blocked?
    A. Sympathetic, proprioception, pain, motor
    B. Sympathetic, pain, proprioception, motor
    C. Motor, pain, proprioception, sympathetic
    D. Pain, proprioception, sympathetic, motor
    E. Pain, proprioception, motor, sympathetic
A
  1. (B) Differential nerve blockade is a complex process where anatomic and chemical factors determine the susceptibility
    of fibers to blockade by local anesthetics. Diameter, myelinization, and location within the nerve trunk affect
    the onset and regression time. In general, the small unmyelinated sympathetic fibers are blocked first, followed
    by unmyelinated C fibers (pain and temp), then small myelinated fibers (proprioception, touch, pressure), and
    finally the large myelinated fibers (motor) (Barash: Clinical Anesthesia, ed 5, pp 456-457, 708; Cousins: Neural
    Blockade in Clinical Anesthesia and Management of Pain, ed 3, pp 45-46).
14
Q
  1. A 95-year-old woman has persistent and prolonged thoracic pain after a herpes zoster infection. Which of the treatments
    below would be the LEAST efficacious in the treatment of her pain?
    A. Oral amitriptyline
    B. Oral clonidine
    C. Topical capsaicin ointment
    D. Transcutaneous electrical nerve stimulation (TENS)
    E. Topical lidocaine patch
A
  1. (B) Acute herpes zoster is due to the reactivation of the varicella-zoster virus. Acute treatment includes symptomatic
    pain treatment and antiviral drugs (e.g., acyclovir). It is typically a benign and self-limiting disease in patients
    younger than 50 years of age. As one gets older, the incidence of postherpetic neuralgia (PHN) defined as pain
    persisting beyond the healing of the herpes zoster lesions increases. The incidence of PHN is about 50% in
    patients older than 50 years. Treatment of established PHN has been shown to be resistant to interventions
    and, thus, can be difficult. However, proven therapies include tricyclic antidepressants, anticonvulsants, topical
    local anesthetics (e.g., 5% lidocaine patch), topical capsaicin and TENS. Sympathetic blocks can provide excellent
    analgesia but are most useful during the more acute stages of the disease rather than during the late chronic
    stages. Sympathetic blocks in the acute stages may decrease the incidence of PHN. Oral clonidine, which is used
    to treat hypertension and opioid withdrawal, has not been shown to be an effective treatment for postherpetic
    neuralgia (Morgan: Clinical Anesthesiology, ed 4, p 407; Raj: Practical Management of Pain, ed 3, pp 187-189).
15
Q
  1. The deep peroneal nerve innervates the
    A. Lateral aspect of the dorsum of the foot
    B. Entire dorsum of the foot
    C. Web space between the great toe and the second toe
    D. Web space between the third and fourth toes
    E. Medial aspect of the dorsum of the foot
A
  1. (C) The deep peroneal nerve innervates the short extensors of the toes and the skin of the web space between the
    great and second toe. The deep peroneal nerve is blocked at the ankle by infiltration between the tendons of the
    anterior tibial and extensor hallucis longus muscle (Brown: Atlas of Regional Anesthesia, ed 3, pp 141-143; Miller:
    Anesthesia, ed 6, pp 1703-1704).
16
Q
803. The correct arrangement of local anesthetics in order of their ability to produce cardiotoxicity from most to least is
A. Bupivacaine, lidocaine, ropivacaine
B. Bupivacaine, ropivacaine, lidocaine
C. Lidocaine, bupivacaine, ropivacaine
D. Ropivacaine, bupivacaine, lidocaine
E. Lidocaine, ropivacaine, bupivacaine
A
  1. (B) Central nervous system (CNS) toxicity from local anesthetics generally parallels anesthetic potency (e.g.,
    bupivacaine is four times as potent as lidocaine, ropivacaine is three times as potent as lidocaine). Cardiovascular
    (CV) toxicity occurs at a higher blood level than CNS toxicity. For bupivacaine and ropivacaine,
    CV toxicity occurs at two times the CNS dose, whereas for lidocaine the CV toxicity occurs at
    seven times the CNS toxicity levels, making lidocaine the least cardiotoxic, and bupivacaine the most
    cardiotoxic of the listed local anesthetics (Barash: Clinical Anesthesia, ed 5, pp 459-467; Stoelting: Basics
    of Anesthesia, ed 5, pp 127-131).
17
Q
  1. Allodynia is defined as
    A. Spontaneous pain in an area or region that is anesthetic
    B. Pain initiated or caused by a primary lesion or dysfunction in the nervous system
    C. An unpleasant abnormal sensation, whether spontaneous or evoked
    D. An increased response to a stimulus that is normally painful
    E. Pain caused by a stimulus that does not normally provoke pain
A
  1. (E) The International Association for the Study of Pain (IASP) has defined several pain terms. Anesthesia dolorosa
    refers to spontaneous pain in an area or region that is anesthetic. Neuropathic pain is pain initiated or caused
    by a primary lesion or dysfunction in the nervous system. Dysesthesia is an unpleasant abnormal sensation,
    whether spontaneous or evoked. Hyperalgesia is an increased response to a stimulus that is normally painful.
    Allodynia is pain caused by a stimulus that does not normally provoke pain (Morgan: Clinical Anesthesia ed 4,
    p 361; Loeser: Bonica’s Management of Pain, ed 3, pp 17-20).
18
Q
805. The primary mechanism by which the action of tetracaine is terminated when used for spinal anesthesia is
A. Systemic absorption
B. Uptake into neurons
C. Hydrolysis by pseudocholinesterase
D. Hydrolysis by nonspecific esterases
E. Spontaneous degradation at 37° C
A
  1. (A) Ester local anesthetics are hydrolyzed by cholinesterase enzymes that are present mainly in plasma and, in a
    smaller amount, in the liver. Because there are no cholinesterase enzymes present in cerebrospinal fluid (CSF),
    the anesthetic effect of tetracaine will persist until it is absorbed into systemic circulation. The rate of hydrolysis
    varies, with chloroprocaine being fastest, procaine intermediate, and tetracaine the slowest. Toxicity is
    inversely related to the rate of hydrolysis; tetracaine is, therefore, the most toxic (Miller: Anesthesia, ed 6, p 589;
    Morgan: Clinical Anesthesiology, ed 4, p 309; Stoelting: Pharmacology and Physiology in Anesthetic Practice, ed 4,
    pp 186-187).
19
Q
  1. Complex regional pain syndrome type I (reflex sympathetic dystrophy) is differentiated from complex regional pain
    syndrome type II (causalgia) by knowledge of its
    A. Etiology
    B. Chronicity
    C. Affected body region
    D. Type of symptoms
    E. Rapidity of onset
A
  1. (A) Complex regional pain syndrome type I or CRPS type I also called reflex sympathetic dystrophy (RSD) is a
    clinical syndrome of continuous burning pain, usually occurring after minor trauma. Patients present with variable
    sensory, motor, autonomic, and trophic changes. Complex regional pain syndrome type II or CRPS type II
    (causalgia) exhibits the same features of reflex sympathetic dystrophy, but the etiology is usually major traumatic
    damage to large nerves (e.g., median nerve of the upper extremity or tibial division of the sciatic nerve in the
    lower extremity) (Barash: Clinical Anesthesia, ed 5, pp 1456-1458; Morgan: Clinical Anesthesia, ed 4, p 406; Raj:
    Practical Management of Pain, ed 3, p 306).
20
Q
807. The primary determinant of local anesthetic potency is
A. pKa
B. Molecular weight
C. Lipid solubility
D. Concentration
E. Protein binding
A
  1. (C) The potency of local anesthetics is directly related to their lipid solubility. In general, the speed or onset of
    action of local anesthetics is related to the pKa of the drug. Drugs with lower pKa values have a higher amount
    of non-ionized molecules at physiologic pH and penetrate the lipid portion of nerves faster (an exception is
    chloroprocaine, which has a fast onset of action that may be related to the higher concentration of drug used).
    The amount of protein binding seems related to the duration of action of local anesthetics (more protein binding
    has longer duration of action) (Raj: Practical Management of Pain, ed 3, pp 560-561; Stoelting: Pharmacology
    and Physiology in Anesthetic Practice, ed 4, pp 180-181).
21
Q
  1. Which of the following would have the greatest effect on the level of sensory blockade after a subarachnoid injection
    of hyperbaric 0.75% bupivacaine?
    A. Coughing during placement of the block
    B. Addition of epinephrine to the local anesthetic solution
    C. Barbotage
    D. Patient weight
    E. Patient position
A
  1. (E) Many factors have an effect on the sensory level after a subarachnoid injection. The baricity of the solution
    and the patient position are the most important determinants of sensory level. The other listed options have
    little to no effect on sensory level. Patient height also has little effect on sensory level (Miller: Anesthesia, ed 6,
    p 1668).
22
Q
  1. Which of the following local anesthetics would produce the lowest concentration in the fetus relative to the maternal
    serum concentration during a continuous lumbar epidural?
    A. Etidocaine
    B. Bupivacaine
    C. Lidocaine
    D. Chloroprocaine
    E. Mepivacaine
A
  1. (D) Chloroprocaine is an ester local anesthetic that is rapidly metabolized by pseudocholinesterase. With the epidural
    injection of chloroprocaine, very little drug is available to cross the placenta, because the half-life is about
    45 seconds (and that which crosses is also rapidly metabolized making fetal effects essentially non-significant).
    The amide local anesthetics undergo liver metabolism and have relatively long half-lives, but with prolonged
    epidural administration may accumulate in the fetus (Barash: Clinical Anesthesia, ed 5, pp 1154-1155; Miller:
    Anesthesia, ed 6, p 2323).
23
Q
  1. Severe hypotension associated with high spinal anesthesia is caused primarily by
    A. Decreased cardiac output secondary to decreased preload
    B. Decreased systemic vascular resistance
    C. Decreased cardiac output secondary to bradycardia
    D. Decreased cardiac output secondary to decreased myocardial contractility
    E. Increased shunting through metarterioles
A
  1. (A) Hypotension with a high spinal anesthesia is related to sympathetic blockade; venodilation (decreases
    preload), arterial dilation (decreases afterload) and a decrease in heart rate (cardioaccelerator fibers
    T1-T4 blockade and a fall in right atrial filling that affects the intrinsic chronotropic stretch receptors).
    With a high spinal, the decrease in venous dilation is the predominant cause of hypotension (Barash: Clinical
    Anesthesia, ed 5, pp 708-709; Miller: Anesthesia, ed 6, pp 1658-1659; Stoelting: Basics of Anesthesia, ed 5,
    pp 259-262).
24
Q
  1. Select the one true statement concerning phantom limb pain.
    A. Most phantom limb pain becomes more severe with time
    B. Most amputees do not experience phantom limb pain
    C. Nerve blocks may be used to treat phantom limb pain
    D. Trauma amputees have a higher incidence of phantom limb pain than nontrauma amputees
    E. The incidence of phantom limb pain increases with more distal amputations
A
  1. (C) The incidence of phantom limb pain is estimated to be 60% to 85%. The incidence of phantom limb pain
    does not differ between traumatic and nontraumatic amputees. The incidence of phantom pain increases with
    more proximal amputation. Although very difficult to treat, nerve blocks are commonly used in an attempt to
    treat phantom pain. These include trigger point injections, peripheral and central nerve blocks, and sympathetic
    blocks (Raj: Practical Management of Pain, ed 3, pp 213-218).
25
Q
812. Which of the following local anesthetics used for intravenous regional anesthesia (Bier block) is most rapidly metabolized
and thus, least toxic?
A. Lidocaine
B. Ropivacaine
C. Mepivacaine
D. Prilocaine
E. Etidocaine
A
  1. (D) Prilocaine is the most rapidly metabolized of the amide local anesthetics and therefore least toxic. 2-Chloroprocaine
    is hydrolyzed rapidly in the blood and, therefore, would appear to be ideal, but it has been associated with
    a high incidence of thrombophlebitis and is therefore not recommended. To avoid toxicity, maximum doses
    are as follows: prilocaine, 3 to 4 mg/kg; lidocaine, 1.5 to 3 mg/kg; ropivacaine, 1.2 to 1.8 mg/kg. Bupivacaine
    is not recommended for Bier blocks because of reports of cardiovascular toxicity and death that have occurred
    (Cousins: Neural Blockade in Clinical Anesthesia and Management of Pain, ed 3, p 400; Miller: Anesthesia, ed 6,
    pp 586-588, 1695).
26
Q
  1. Select the FALSE statement regarding spinal anatomy and spinal anesthesia.
    A. The addition of phenylephrine to lidocaine will prolong spinal anesthesia
    B. A high thoracic sensory block will result in total sympathetic blockade
    C. The largest vertebral interspace is L5-S1
    D. The dural sac extends to the S4-S5 interspace
    E. Tetracaine provides longer anesthesia than does procaine
A
  1. (D) Both phenylephrine and epinephrine will prolong a spinal anesthetic when administering lidocaine. The Taylor
    approach for spinal anesthesia uses a paramedian approach to the L5-S1 interspace—the largest interspace of
    the vertebral column. The sympathetic nervous system originates in the thoracic and lumbar spinal cord T1-L3;
    therefore, a high thoracic sensory level can cause a complete sympathetic block. The dural sac extends to S2-S3,
    not S4-S5. The spinal cord extends to L3 in the infant and L1-L2 in adults (Barash: Clinical Anesthesia, ed 5,
    pp 693-694, 708; Miller: Anesthesia, ed 6, pp 585, 589, 1654-1656).
27
Q
  1. Four days after a left total hip arthroplasty, an obese 62-year-old woman complains of severe back pain in the region
    where the epidural was placed. Over the ensuing 72 hours, the back pain gradually worsens and a severe aching pain
    that radiates down the left leg to the knee develops. The most likely diagnosis is
    A. Epidural abscess
    B. Epidural hematoma
    C. Anterior spinal artery syndrome
    D. Arachnoiditis
    E. Meralgia paresthetica
A
  1. (A) Development of an epidural abscess is fortunately an exceedingly rare complication of spinal and epidural
    anesthesia. Most anesthetic related epidural abscesses are associated with epidural catheters. When an epidural
    abscess is developing, prompt recognition and treatment are essential if permanent sequelae are to be avoided.
    Symptoms from an epidural abscess may not become apparent until several days (mean 5 days) after placement
    of the block. There are four clinical stages of epidural abscess symptom progression. Initially, localized back
    pain develops. Second stage includes nerve root or radicular pain. The third stage involves motor and sensory
    deficits followed by the last stage of paraplegia. Unlike an epidural hematoma, in which severe back pain is the
    key feature, patients with epidural abscesses will complain of radicular pain approximately 3 days after development
    of the back pain. Anterior spinal artery syndrome is characterized predominantly by motor weakness or
    paralysis of the lower extremities. Meralgia paresthetica is related to entrapment of the lateral femoral cutaneous
    nerve as it courses below the inguinal ligament and is associated with burning pain over the lateral aspect of the
    thigh. It is not a complication of epidural anesthesia (Morgan: Clinical Anesthesia, ed 4, pp 320-321; Raj: Practical
    Management of Pain, ed 3, p 649).
28
Q
  1. Which of the following choices is NOT consistent
    with a limb affected by complex regional pain syndrome?
    A. Osteoporosis
    B. Allodynia
    C. Dermatomal distribution of pain
    D. Atrophy of the involved extremity
    E. Hyperesthesia
A
  1. (C) Complex regional pain syndromes are associated with trauma. The main feature is burning and continuous pain
    that is exacerbated by normal movement, cutaneous stimulation, or stress, usually weeks after the injury. The
    pain is not anatomically distributed. Other associated features include cool, red, clammy skin and hair loss in
    the involved extremity. Chronic cases may be associated with atrophy and osteoporosis (Barash: Clinical Anesthesia,
    ed 5, pp 1456-1458; Miller: Anesthesia, ed 6, pp 2774-2775).
29
Q
  1. The main advantage of neurolytic nerve blockade with phenol versus alcohol is
    A. Denser blockade
    B. Blockade is permanent
    C. The effects of the block can be evaluated immediately
    D. The block is less painful
    E. Phenol is selective for sympathetic fibers
A
  1. (D) Neurolytic blockade with phenol (6% to 10% in glycerine) is painless because phenol has a dual action as
    both a local anesthetic and a neurolytic agent. The initial block wears off over a 24-hour period, during which
    time neurolysis occurs. For this reason you must wait a day to determine effectiveness of the neurolytic block.
    Alcohol (100% ethanol) is painful on injection and should be preceded by local anesthetic injection. Unfortunately,
    there is no neurolytic agent that affects only sympathetic fibers (Barash: Clinical Anesthesia, ed 5,
    p 1464).
30
Q
817. How much local anesthetic should be administered per spinal segment to patients between 20 and 40 years of age
receiving a lumbar epidural anesthetic?
A. 0.25 to 0.5 mL
B. 0.5 to 1.0 mL
C. 1 to 2 mL
D. 2 to 3 mL
E. 3 to 5 mL
A
  1. (C) In general, each 1-2 mL of local anesthetic will anesthetize about one spinal segment in the 20 to 40-year-old
    patient. Because of the negative intrathoracic pressure transmitted to the epidural space with breathing, about
    two thirds of the segments are blocked above the level of the lumbar placement and one third of segments are
    blocked below the injection. For example, to achieve a T4 block when an epidural is placed at the L2-L3 space
    about 10 segments above and 5 segments below the epidural would be needed (15 segments) or about 20-25 mL.
    As one gets older, the dose of local anesthetic mL/segment decreases (e.g., 80 year old may need 0.75-1.5 mL/
    segment). Also, pregnant patients are more sensitive to local anesthetics and reduced doses are needed (Barash:
    Clinical Anesthesia, ed 5, pp 705-706; Morgan: Clinical Anesthesiology, ed 4, p 312; Stoelting: Basics of Anesthesia,
    ed 5, p 266).
31
Q
818. The artery of Adamkiewicz most frequently arises from the aorta at which spinal level?
A. T1-T4
B. T5-T8
C. T9-T12
D. L1-L4
E. L5-S3
A
  1. (C)
    The artery of Adamkiewicz is also called the arteria radicularis magna and is one of the “feeder” arteries for the
    anterior spinal artery. Damage to this artery can lead to ischemia in the thoracolumbar region and may result in
    paraplegia. The origin of this artery is variable (e.g., T9-T12 in 75% of cases, L1-L2 in 10% of cases) (Barash:
    Clinical Anesthesia, ed 5, p 958; Miller: Anesthesia, ed 6, p 2087).
32
Q
  1. The anterior and posterior spinal arteries originate from the
    A. Common carotid and vertebral arteries, respectively
    B. Internal carotid and vertebral arteries, respectively
    C. Internal carotid and posterior cerebral arteries, respectively
    D. Vertebral and anterior cerebellar arteries, respectively
    E. Vertebral, radicular arteries and the posterior inferior cerebellar arteries, respectively
A
  1. (E) The one anterior spinal artery supplies about 75% of the blood flow to the spinal cord (motor tracts) and arises
    from the vertebral arteries and radicular arteries from the aorta. It descends in front of the anterior longitudinal
    sulcus of the spinal cord. The two posterior spinal arteries supply about 25% of the blood flow to the spinal
    cord (sensory tracts) and arise from the posterior and inferior cerebellar arteries, the vertebral arteries, and the
    radicular arteries (Barash: Clinical Anesthesia, ed 5, p 958; Miller: Anesthesia, ed 6, pp 2086-2087).
33
Q
  1. Important landmarks for performing a sciatic nerve block (classic approach of Labat) include
    A. Iliac crest, sacral hiatus, greater trochanter
    B. Iliac crest, coccyx, and greater trochanter
    C. Posterior superior iliac spine, coccyx, and greater trochanter
    D. Posterior superior iliac spine, greater trochanter and sacral hiatus
    E. Posterior superior iliac spine and greater trochanter
A
  1. (D) To perform a sciatic nerve block, first draw a line from the posterior superior iliac spine to the greater trochanter,
    then draw a 5-cm line perpendicular from the midpoint of this line caudally and a second line from the
    sacral hiatus to the greater trochanter. The intersection of the second line with the perpendicular line marks the
    point of entry (Brown: Atlas of Regional Anesthesia, ed 3, pp 105-110; Miller: Anesthesia, ed 6, p 1700).
34
Q
  1. A 36-year-old female patient is undergoing thyroidectomy under a deep cervical plexus nerve block. Which of the
    following complications would be LEAST likely with this block?
    A. Horner’s syndrome
    B. Subarachnoid injection
    C. Blockade of the recurrent laryngeal nerve
    D. Blockade of the spinal accessory nerve
    E. Vertebral artery injection
A
  1. (D) Complications of deep cervical plexus block include injection of the local anesthetic into the vertebral artery,
    subarachnoid space, or epidural space. Other nerves that may be anesthetized include the phrenic nerve (which
    is why bilateral deep cervical plexus blocks should be performed with caution, if at all), and the recurrent laryngeal
    nerve (Barash: Clinical Anesthesia, ed 5, p 723; Brown: Atlas of Regional Anesthesia, ed 3, pp 191-195; Miller:
    Anesthesia, ed 6, p 1707).
35
Q
  1. A retrobulbar block anesthetizes each of the following nerves EXCEPT
    A. Ciliary nerves
    B. Cranial nerve IV (trochlear nerve)
    C. Cranial nerve III (oculomotor nerve)
    D. Cranial nerve VI (abducens nerve)
    E. Maxillary branch of the trigeminal nerve
A
  1. (E) A retrobulbar block anesthetizes the three cranial nerves responsible for movement of the eye. The ciliary nerves
    are also blocked, providing anesthesia to the conjunctiva, cornea, and uvea. The ophthalmic branch of the trigeminal
    nerve provides sensory innervation to the skin of the forehead, cornea, and eyelid. This branch of the
    trigeminal nerve may be blocked, but the maxillary branch would be spared (Barash: Clinical Anesthesia, ed 5,
    pp 984-986; Brown: Atlas of Regional Anesthesia, ed 3, pp 185-188).
36
Q
823. Which of the following muscles of the larynx is innervated by the external branch of the superior laryngeal nerve?
A. Vocalis muscle
B. Thyroarytenoid muscles
C. Posterior cricoarytenoid muscle
D. Oblique arytenoid muscles
E. Cricothyroid muscle
A
  1. (E) The vagus nerve innervates the airway by two branches: the superior laryngeal nerves and the recurrent laryngeal
    nerves. All the muscles of the larynx are innervated by the recurrent laryngeal nerve except for the cricothyroid
    muscle. The superior laryngeal nerve divides into the internal and external laryngeal branches. The external laryngeal branch innervates the cricothyroid muscle. The internal laryngeal branch provides sensory fibers to the
    cords, epiglottis and the arytenoids (Barash: Clinical Anesthesia, ed 5, p 724; Brown: Atlas of Regional Anesthesia,
    ed 3, pp 207-211).
37
Q
824. All the following agents are acceptable for use in a Bier block EXCEPT
A. 0.5% Lidocaine
B. 0.5% Mepivacaine
C. 0.5% Procaine
D. 0.5% Prilocaine
E. 0.25% Bupivacaine
A
  1. (E) Because of the potential for cardiotoxicity and because bupivacaine has no advantages over other local anesthetics
    in this setting, it is no longer recommended for use in intravenous regional anesthesia (Miller: Anesthesia,
    ed 6, p 1695).
38
Q
825. The stellate ganglion lies in closest proximity to which of the following vascular structures?
A. Common carotid artery
B. Internal carotid artery
C. Vertebral artery
D. Axillary artery
E. Aorta
A
  1. (C)
    The stellate ganglion usually lies in front of the neck of the first rib. The vertebral artery lies anterior to the ganglion
    as it has just originated from the subclavian artery. After passing over the ganglion, it enters the vertebral
    foramen and lies posterior to the anterior tubercle of C6 (Barash: Clinical Anesthesia, ed 5, pp 736-737; Brown:
    Atlas of Regional Anesthesia, ed 3, pp 199-203; Raj: Practical Management of Pain, ed 3, pp 655-657).
39
Q
826. Which of the following structures in the antecubital fossa is the most medial?
A. Brachial artery
B. Radial nerve
C. Tendon of the biceps
D. Median nerve
E. Musculocutaneous nerve
A
  1. (D) The median nerve is the most medial structure in the antecubital fossa. To block this nerve, first the brachial
    artery is palpated at the level of the intercondylar line between the medial and lateral epicondyles, and then a
    needle is inserted just medial to the artery and directed perpendicularly to the skin (Brown: Atlas of Regional
    Anesthesia, ed 3, pp 73-74; Morgan: Clinical Anesthesia, ed 4, pp 338-339).
40
Q
  1. During placement of an epidural in a 78-year-old patient scheduled for a total knee arthroplasty, the patient complains
    of a sharp sustained pain radiating down his left leg as the catheter is inserted to 2 cm. The most appropriate
    action at this time would be
    A. Leave the catheter at 2 cm, give test dose
    B. Give small dose to relieve pain then advance 1 cm
    C. Withdraw the catheter 1 cm, give test dose
    D. Withdraw needle and catheter, reinsert in a new position
    E. Abandon epidural technique, place long-acting spinal anesthetic
A
  1. (D) When an epidural catheter is placed without fluoroscopic guidance, the exact location of the needle tip relative
    to the anatomic structures of the back can only be surmised. If malposition of either the needle or the catheter
    is suspected, it is prudent to withdraw the entire apparatus and reinsert a second time. In this case, it is possible
    that the catheter tip has found its way into a nerve root. Under these circumstances, injection of a local anesthetic
    or narcotic could produce pressure that would lead to ischemia and possible neurologic damage. During
    placement or injection of an epidural catheter, a paresthesia is always a warning sign that should be heeded (Raj:
    Practical Management of Pain, ed 3, p 650).
41
Q
828. Cutaneous innervation of the plantar surface of the foot is provided by the
A. Sural nerve
B. Posterior tibial nerve
C. Saphenous nerve
D. Deep peroneal nerve
E. Superficial peroneal nerve
A
  1. (B) There are five nerves that supply the ankle and foot: the posterior tibial, sural, superficial and deep peroneal,
    and saphenous nerves. These nerves are superficial at the level of the ankle and are easy to block. The posterior
    branch of the tibial nerve gives rise to the medial and lateral plantar nerves, which supply the plantar
    surface of the foot (Barash: Clinical Anesthesia, ed 5, pp 743-744; Brown: Atlas of Regional Anesthesia, ed 3,
    pp 141-143).
42
Q
  1. Which of the following local anesthetics has the lowest ratio of dosage required for cardiovascular collapse to dosage
    required for central nervous system toxicity?
    A. Lidocaine
    B. Etidocaine
    C. Bupivacaine
    D. Prilocaine
    E. Chloroprocaine
A
  1. (C) In general, in both in vivo and in vitro studies there is an overall direct correlation between anesthetic’s potency
    and its direct depressant effect on myocardial contractility. The ratio of dosage required for cardiovascular collapse
    in animal models compared with that required to produce neurologic symptoms is the lowest for bupivacaine,
    levo-bupivacaine and ropivacaine (2.0). Ratios for other local anesthetics are as follows: prilocaine, 3.1;
    procaine and chloroprocaine, 3.7; etidocaine, 4.4; lidocaine and mepivacaine, 7.1 (Barash: Clinical Anesthesia,
    ed 5, pp 462-466).
43
Q
  1. A 57-year-old patient is scheduled for hemorrhoidectomy. The patient has a history of mild chronic obstructive pulmonary
    disease, hypertension, and traumatic foot amputation from a tractor accident. His only hospitalizations were
    for two suicide attempts related to phantom limb sensations 10 years ago. He takes phenelzine (Nardil), thiazide, and
    potassium. Which of the following anesthetic techniques would be most appropriate for this patient?
    A. Spinal anesthetic with 0.5% hyperbaric bupivacaine
    B. Epidural anesthetic with 0.5% bupivacaine
    C. Local infiltration with lidocaine and epinephrine, sedation with propofol and meperidine
    D. General anesthesia with thiopental sodium (Pentothal), succinylcholine, nitrous oxide, isoflurane, meperidine
    E. General anesthesia with propofol, succinylcholine, nitrous oxide, fentanyl
A
  1. (E) Reactivation of phantom limb sensations has been reported in patients who have received both spinal and
    epidural anesthetics (90% in some series). In the majority of these cases (80%), phantom limb sensation persisted
    until the block receded. With a history of a phantom limb sensations that drove this patient to attempt
    suicide, it is probably wise to avoid spinal and epidural anesthetics. Phenelzine (Nardil) is a monoamine oxidase
    (MAO) inhibitor that is occasionally used for the treatment of depression. Any anesthetic or combination of
    techniques that involves meperidine is contraindicated in patients receiving MAO inhibitors. The combination
    of meperidine and MAO inhibitors has been associated with hyperthermia, hypotension, hypertension, ventilatory
    depression, skeletal muscle rigidity, seizures, and coma. Because of this unfavorable drug interaction,
    meperidine should be avoided in patients receiving MAO inhibitors. Accordingly, the only acceptable choice
    in this question would be general anesthesia with propofol, succinylcholine, nitrous oxide, and fentanyl. As an
    interesting side point, the drug phenelzine prolongs the duration of action of succinylcholine by decreasing
    plasma cholinesterase activity (Miller: Anesthesia, ed 6, pp 423-424; Morgan: Clinical Anesthesiology, ed 4, p 657;
    Raj: Practical Management of Pain, ed 3, p 212).
44
Q
  1. If the recurrent laryngeal nerve were transected bilaterally, the vocal cords would
    A. Be in the open position
    B. Be in the closed position
    C. Be in the intermediate position (i.e., 2-3 mm apart)
    D. Not be affected unless the superior laryngeal nerve were also injured
    E. Appear exactly the same as if an intubating dose of succinylcholine were given
A
  1. (C) The recurrent laryngeal nerve innervates all the muscles of the larynx except the cricothyroid muscle, which
    tenses the vocal cords and is innervated by the external branch of the superior laryngeal nerve. With bilateral
    transections of the recurrent laryngeal nerve, the vocal cords lie within 2 to 3 mm of the midline. The airway
    maybe inadequate and a tracheostomy may be needed (Miller: Anesthesia, ed 6, p 2538).
45
Q
832. A 63-year-old woman undergoes total knee arthroplasty under spinal anesthesia. Two days later she complains of a
severe headache on the left side of her head. Pain intensity is not related to posture. The LEAST likely cause of this
headache is
A. Caffeine withdrawal
B. Malingering
C. Viral illness
D. Migraine
E. Postdural puncture headache
A
  1. (E) Postdural puncture headache (PDPH) will have a postural component. When supine, the headache is usually
    gone but may be mild in some cases. When the head is elevated the headache may be severe, is bilateral and
    may be associated with diplopia, nausea and vomiting. The headache pain is typically frontal and/or occipital in
    location (Barash: Clinical Anesthesia, ed 5, p 711; Stoelting: Basics of Anesthesia, ed 5, p 260).
46
Q
833. What is the correct order of structures (from cephalad to caudad) in the intercostal space?
A. Nerve, artery, vein
B. Vein, nerve, artery
C. Vein, artery, nerve
D. Artery, nerve, vein
E. Artery, vein, nerve
A
  1. (C) VAN (Vein, Artery, Nerve) describes the anatomical relationship of the intercostal structures deep to the lower
    border of the ribs from cephalad to caudal direction. The block is performed by walking off the inferior edge
    of the rib typically about 5 to 7 cm from midline. The two principle risks are pneumothorax and intravascular
    injection of local anesthetics. Because of the close proximity of the vein and artery to the nerve, intercostal
    blocks have relatively high blood levels as compared to other blocks (e.g., epidural, brachial plexus, infiltration)
    and caution with dose is needed if many levels are blocked (Morgan: Clinical Anesthesiology, ed 4, pp 353-354;
    Stoelting: Basics of Anesthesia, ed 5, pp 282-283).
47
Q
834. Which of the following types of regional anesthesia is associated with the greatest serum concentration of lidocaine?
A. Intercostal
B. Caudal
C. Epidural
D. Brachial plexus
E. Femoral nerve block
A
  1. (A) The site of injection of the local anesthetic is one of the most important factors influencing systemic local anesthetic
    absorption and toxicity. The degree of absorption from the site of injection depends on the blood supply
    to that site. Areas that have the greatest blood supply have the greatest systemic absorption. For this reason, the
    greatest plasma concentration of local anesthetic occurs after an intercostal block, followed by caudal epidural,
    lumbar epidural, brachial plexus, sciatic/femoral nerve block, and subcutaneous (Barash: Clinical Anesthesia,
    ed 5, pp 460-461; Miller: Anesthesia, ed 6, p 591; Stoelting: Basics of Anesthesia, ed 5, p 130).
48
Q
  1. Differences in which of the following local anesthetic properties account for the fact that the onset of an epidural block
    with 3% 2-chloroprocaine is more rapid than 2% lidocaine?
    A. Protein binding
    B. pKa
    C. Lipid solubility
    D. Concentration
    E. Ester versus amide structure
A
  1. (D) Local anesthetics are weak bases. The neutral (non-ionized) form of the molecule is able to pass through the
    lipid nerve cell membrane, whereas the ionized (protonated) form actually produces anesthesia. Chloroprocaine
    has the highest pKa of local anesthetics, meaning that a greater percentage of it will exist in the ionized
    form at any given pH than any of the other local anesthetics. Despite this fact, 3% chloroprocaine has a more
    rapid onset than 2% lidocaine, presumably because of the greater number of molecules (concentration). If one
    compares onset time for 1.5% lidocaine against 1.5% chloroprocaine, the former will have a more rapid onset
    (Miller: Anesthesia, ed 6, p 584).
49
Q
  1. A 69-year-old man with a history of diabetes mellitus and chronic renal failure is to undergo placement of a dialysis
    fistula under regional anesthesia. During needle manipulation for a supraclavicular brachial plexus block, the patient
    begins to cough and complain of chest pain and shortness of breath. The most likely diagnosis is
    A. Angina
    B. Pneumothorax
    C. Phrenic nerve irritation
    D. Intravascular injection of local anesthetic
    E. Intrathecal injection of local anesthetic
A
  1. (B) The risk of pneumothorax is a significant limitation for supraclavicular brachial plexus blocks (incidence 0.5%-6%
    depending upon experience). Furthermore, the technique is difficult to teach and describe. For these reasons,
    this block should not be performed in patients in whom a pneumothorax or phrenic nerve block (40%-60% of patients) would result in significant dyspnea or respiratory distress. A pneumothorax should be considered
    if the patient begins to complain of chest pain or shortness of breath or begins to cough during placement of
    supraclavicular brachial plexus block (Barash: Clinical Anesthesia, ed 5, p 728; Miller: Anesthesia, ed 6, p 1690).
50
Q
837. Which of the following nerves is located immediately lateral to the trachea?
A. Vagus
B. Recurrent laryngeal
C. Phrenic
D. Long thoracic
E. Spinal accessory
A
  1. (B) The structures in the neck from medial to lateral are the recurrent laryngeal nerve, carotid artery, vagus nerve,
    internal jugular vein, and phrenic nerve (Brown: Atlas of Regional Anesthesia, ed 3, pp 28, 33, 208-209; Clemente:
    Anatomy: Regional Atlas of the Human Body, ed 3, p 586).
51
Q
838. If a needle is introduced 1.5 cm inferior and 1.5 cm lateral to the pubic tubercle, to which nerve will it lie in close
proximity?
A. Obturator nerve
B. Femoral nerve
C. Lateral femoral cutaneous nerve
D. Sciatic nerve
E. Ilioinguinal nerve
A
  1. (A) The obturator nerve provides variable cutaneous innervation of the thigh. An obturator nerve block is achieved
    by placement of the needle 1 to 2 cm lateral to and 1 to 2 cm below the pubic tubercle. After contact with
    the pubic bone, the needle is withdrawn and walked cephalad to identify the obturator canal. Between 10 and
    15 mL of local anesthetic should be placed in the canal. If a nerve stimulator is used, contraction of the adductor
    muscles with nerve stimulation indicates proximity to the nerve (Stoelting: Basics of Anesthesia, ed 5, p 285).
52
Q
  1. The most common complication associated with a supraclavicular brachial plexus block is
    A. Blockade of the phrenic nerve
    B. Intravascular injection into the vertebral artery
    C. Spinal blockade
    D. Blockade of the recurrent laryngeal nerve
    E. Pneumothorax
A
  1. (A) The most serious complication associated with a supraclavicular brachial plexus block is pneumothorax. The
    most common complication is a phrenic nerve block which is usually mild and relatively common (40%-60% of
    blocks). Bilateral supraclavicular blocks however, are not recommended due to the possibility of bilateral phrenic
    nerve paralysis or pneumothoraces. Other potential complications include Horner’s syndrome, nerve damage or
    neuritis, or intravascular injection (Miller: Anesthesia, ed 6, p 1690; Stoelting: Basics of Anesthesia, ed 5, p 278).
53
Q
840. Which portion of the upper extremity is not innervated by the brachial plexus?
A. Posterior medial portion of the arm
B. Elbow
C. Lateral portion of the forearm
D. Medial portion of the forearm
E. Anterolateral portion of the arm
A
  1. (A) The arm receives sensory innervation from the brachial plexus except for the shoulder, which is innervated by
    the cervical plexus, and the posterior medial aspect of the arm, which is supplied by the intercostobrachial nerve
    (Brown: Atlas of Regional Anesthesia, ed 3, pp 27-31; Stoelting: Basics of Anesthesia, ed 5, p 277).
54
Q
841. Which section of the brachial plexus is blocked with a supraclavicular block?
A. Roots
B. Trunks
C. Divisions
D. Cords
E. Branches
A
  1. (B) The brachial plexus starts out at the root level from the ventral rami of C5-T1 with a small amount from C4 and
    T2. These roots at the level of the scalene muscle become the 3 trunks: superior, middle and inferior. The trunks
    then divide into the dorsal and ventral divisions at the lateral edge of the first rib. When the divisions enter the
    axilla, they become the cords: posterior, lateral and medial. At the lateral border of the pectoralis muscle they
    become the five peripheral nerves: radial, musculocutaneous, median, ulnar and axillary. The plexus is blocked
    at the distal level of the trunks just before they become divisions. Here a small volume of anesthetic is required
    and no part of the plexus is spared, as with axillary or interscalene block. The block can be performed with the
    arm in any position (Brown: Atlas of Regional Anesthesia, ed 3, pp 27-35, 47-54; Cousins: Neural Blockade in
    Clinical Anesthesia and Management of Pain, ed 3, p 352; Miller: Anesthesia, ed 6, p 1689).
55
Q
842. A celiac-plexus block would NOT effectively treat pain resulting from a malignancy involving which of the following
organs?
A. Uterus
B. Adrenal gland
C. Stomach
D. Pancreas
E. Gallbladder
A
  1. (A) The celiac plexus innervates most of the abdominal viscera, including the lower esophagus, stomach, all of the
    small intestine and the large intestine up to the splenic flexure as well as the pancreas, liver, biliary tract, spleen,
    kidneys, adrenal glands and omentum. The pelvic organs (e.g., uterus, ovaries, prostate, distal colon) are supplied
    by the hypogastric plexus (Brown: Atlas of Regional Anesthesia, ed 3, pp 313, 325).
56
Q
  1. A healthy 27-year-old female stepped on a nail and is to undergo débridement of a wound on her right great toe. She
    is anxious about general anesthesia but agrees to an ankle block with mild sedation. Which nerves must be adequately
    blocked in order to perform the surgery?
    A. Deep peroneal, posterior tibial, saphenous, sural
    B. Deep peroneal, saphenous, superficial peroneal, sural
    C. Deep peroneal, posterior tibial, superficial peroneal, sural
    D. Deep peroneal, posterior tibial, saphenous, superficial peroneal
    E. Deep peroneal, posterior tibial, saphenous
A
  1. (D) The great toe is innervated by the deep peroneal, posterior tibial, superficial peroneal, and occasionally the
    saphenous nerve. All four of these nerves should be blocked for surgery on the great toe (Miller: Anesthesia,
    ed 6, pp 1703-1704; Stoelting: Basics of Anesthesia, ed 5, p 288).
57
Q
  1. A 54-year-old man is administered morphine via patient-controlled analgesia (PCA) pump after a left total hip arthroplasty.
    The pump is programmed to deliver a maximum dose of 2 mg every 15 minutes (lockout time) as needed
    for patient comfort. The total maximum dose that can be delivered in 4 hours is 30 mg. On the first day the patient
    receives 15 doses every 4 hours by pressing the delivery button every 15 to 18 minutes. How should his pain control
    be further managed?
    A. Discontinue the PCA pump and administer intramuscular morphine
    B. Increase the lockout time from 15 to 25 minutes
    C. Change the analgesic from morphine to fentanyl
    D. Increase the dose to 3 mg every 15 minutes as needed up to a total maximum dose of 40 mg every 4 hours
    E. Make no changes
A
  1. (D) Frequent dosing by a patient receiving postoperative analgesia through a PCA pump suggests the need to
    increase the magnitude of the dose. It is important to keep in mind that a patient should be given a sufficient
    loading dose of narcotic before initiative therapy with a PCA pump. Otherwise, the patient will be playing the
    frustrating game of “catch up” (Barash: Clinical Anesthesia, ed 5, p 1421).
58
Q
  1. The mechanism of the TENS unit in relieving pain is
    A. Direct electrical inhibition of type A-delta and C fibers
    B. Depletion of neurotransmitter in nociceptors
    C. Hyperpolarization of spinothalamic tract neurons
    D. Activation of inhibitory neurons
    E. Distortion of nociceptors
A
845. (D) Transcutaneous electrical nerve stimulation (TENS) is low-intensity electrical stimuli (2 and 100 Hz,) that
produces a tingling or vibratory sensation. It is thought that TENS units produce analgesia by releasing endogenous
endorphins. These endorphins have an inhibitory effect at the spinal cord level and augment descending
inhibitory pathways (Miller: Anesthesia, ed 6, pp 2776-2777; Stoelting: Basics of Anesthesia, ed 5, p 591).
59
Q
846. Epidural use of which of the following opioids would result in the greatest incidence of delayed respiratory depression?
A. Sufentanil
B. Fentanyl
C. Morphine sulfate
D. Hydromorphone
E. Methadone
A
  1. (C) Although the more hydrophilic drugs such as morphine have a longer duration of action of analgesia, they
    also have a higher potential for inducing delayed respiratory depression through cephalad migration in the
    CNS, as compared with the more lipid-soluble drugs listed in this question (Barash: Clinical Anesthesia, ed 5,
    pp 1426-1428; Miller: Anesthesia, ed 6, pp 1472-1474).
60
Q
847. A 21-year-old patient reports tingling in her thumb during cesarean section under epidural anesthesia. To which dermatomal
level would this correspond?
A. C4
B. C5
C. C6
D. C7
E. C8
A
  1. (C) The thumb corresponds to dermatome C6, the second and middle fingers correspond to dermatome C7, and
    the fourth and little fingers correspond to dermatome C8 (Stoelting: Basics of Anesthesia, ed 5, p 249).
61
Q
  1. Which of the following would hasten the onset and increase the clinical duration of action of a local anesthetic, and
    provide the greatest depth of motor and sensory blockade when used for epidural anesthesia?
    A. Addition to 1:200,000 epinephrine
    B. Increasing the volume of local anesthetic
    C. Increasing the concentration of local anesthetic
    D. Increasing the dose
    E. Placing the patient in the head-down position
A
  1. (D) Increasing the total dose (mass) of local anesthetic is more efficacious in hastening the onset and increasing the
    duration of an epidural anesthetic than increasing the volume or increasing the concentration (while holding
    the total dose constant) (Barash: Clinical Anesthesia, ed 5, p 707).
62
Q
  1. Select the FALSE statement concerning neurolytic nerve blocks.
    A. There is little difference in the efficacy between alcohol and phenol
    B. Destruction of peripheral nerves can be followed by a denervation hypersensitivity that is worse than the original pain
    C. Neurolytic blocks should be reserved for patients with short life expectancies
    D. Neurolytic blockade with phenol is permanent
    E. Intrathecal neurolysis may be an effective management for certain pain conditions
A
  1. (D) Alcohol and phenol are similar in their ability to cause nonselective damage to neural tissues. Alcohol causes
    pain when injected and sometimes is mixed with bupivacaine, whereas phenol is relatively painless. Neural tissue
    will regenerate; therefore, neurolytic blocks are never “permanent” and neurolysis can lead to a denervation
    hypersensitivity, which can be extremely painful (Barash: Clinical Anesthesia, ed 5, p 1464; Morgan: Clinical
    Anesthesiology, ed 4, pp 388-389).
63
Q
  1. The addition of epinephrine to epidural bupivacaine will
    A. Prolong motor blockade only
    B. Prolong sensory blockade only
    C. Prolong motor and sensory blockade
    D. Shorten duration of sensory blockade
    E. Have no effect on either duration of motor or sensory blockade
A
  1. (B) Epinephrine’s effect on the duration of anesthesia depends on the local anesthetic and the site. Infiltration and
    peripheral block duration with most agents will be prolonged with epinephrine. The addition of epinephrine to
    epidural 0.5% or 0.75% bupivacaine has not been shown to increase the duration of the motor blockade but does
    extend the duration of the sensory block. The effect of epinephrine is greater for the intermediate duration local anesthetics
    lidocaine and mepivacaine (Barash: Clinical Anesthesia, ed 5, p 707; Miller: Anesthesia, ed 6, pp 1674-1675).
64
Q
  1. The epidural administration of a mixture of chloroprocaine and bupivacaine would have
    A. A latency similar to chloroprocaine with a duration of action similar to bupivacaine
    B. A latency shorter than chloroprocaine with a duration of action longer than bupivacaine
    C. A latency shorter than chloroprocaine with a duration of action similar to bupivacaine
    D. A latency longer than chloroprocaine with a duration of action similar to chloroprocaine
    E. A latency longer than chloroprocaine with a duration of action shorter than bupivacaine
A
  1. (E) Mixtures of local anesthetics have been used to take advantage of the short latency of certain agents (chloroprocaine)
    and the long duration of other agents (bupivacaine). Duration of epidural anesthesia by mixtures of
    chloroprocaine and bupivacaine has been shown to be shorter than bupivacaine alone and onset time longer
    than chloroprocaine alone (Miller: Anesthesia, ed 6, pp 585-586).
65
Q
852. Each of the following is associated with an increased incidence postdural puncture headaches (PDPH) EXCEPT
A. Young age
B. Female gender
C. Early ambulation
D. Pregnancy
E. Large needle size
A
  1. (C) Younger adults have a higher incidence of PDPH than older adults. Women have a slightly higher incidence
    than men. Pregnant women have a higher incidence than nonpregnant women. Since the incidence and severity
    of PDPH relates to the amount of CSF leakage through the dural hole, it makes sense that the larger the needle
    and the more holes in the dura the greater incidence of PDPH. In addition, the shape of the tip of the needle
    is important; a cutting needle (e.g.. Quincke) has a greater incidence of PDPH than noncutting needles (e.g.,
    Whitacre, Sprotte). The incidence of headache has been shown to be less when the dural fibers are split longitudinally
    rather than when they are cut while the needle is held in a transverse direction. The timing of ambulation
    relative to dural puncture has not been shown to affect the incidence of postspinal headache. The block should
    wear off before ambulation is attempted (Barash: Clinical Anesthesia, ed 5, p 711; Miller: Anesthesia, ed 6, p 1669).
66
Q
  1. Each of the following items describes pain in the abdominal viscera EXCEPT
    A. Pain is transmitted via the vagus nerve
    B. The nerve fibers are type C
    C. Pain is not in a dermatomal distribution
    D. Pain is characterized by a dull aching or burning sensation
    E. Distention of the transverse colon causes more pain than surgical transection
A
  1. (A) Virtually all pain arising in the thoracic or abdominal viscera is transmitted via the sympathetic nervous system
    in unmyelinated type C fibers. Visceral pain is dull, aching, burning, and nonspecific. Visceral pain is caused by
    any stimulus that excites nociceptive nerve endings in diffuse areas. In this regard, distention of a hollow viscus
    causes a greater sensation of pain than does the highly localized damage produced by transecting the gut (Raj:
    Practical Management of Pain, ed 3, pp 223-225).
67
Q
  1. A 24-year-old man undergoes repair of a right anterior shoulder dislocation under interscalene brachial plexus block.
    Anesthesia is produced with 30 mL of 0.5% bupivacaine with 5 μg/mL of epinephrine. The next morning, the patient
    complains of numbness in his right arm and hand. The most likely cause of these complaints is
    A. Excessive retraction by the surgeon
    B. Prolonged pressure on the brachial plexus from malpositioning
    C. Pressure on the right medial epicondyle from malpositioning
    D. Pressure on the right posterior humerus from malpositioning
    E. Residual anesthesia
A
  1. (E) The brachial plexus is not normally retracted during repair of an anterior shoulder dislocation. Prolonged
    pressure on the brachial plexus will result in hand or arm numbness. This may occur if this structure becomes
    pinched between the clavicle and the head of the humerus, as seen in patients placed in steep Trendelenburg
    position with the shoulders resting against shoulder braces. Prolonged pressure on the medial epicondyle may
    produce an ulnar neuropathy, whereas prolonged pressure against the posterior surface of the humerus may
    produce a radial neuropathy. Bupivacaine is a long-acting local anesthetic and may cause numbness for 8 to 12
    hours (Brown: Atlas of Regional Anesthesia, ed 3, p 39).
68
Q
  1. Which of the following patients would be LEAST likely to develop a decrease in heart rate with a high (C8) level
    spinal anesthesia?
    A. A 15-year-old female patient with history of Wolff-Parkinson-White syndrome
    B. A 73-year-old patient with glaucoma treated with pilocarpine eye drops
    C. A 33 year old with a T6 paraplegia
    D. A 45-year-old diabetic man with a history orthostatic hypotension
    E. A 47-year-old patient who had a myocardial infarction 1 month ago, now taking procainamide
A
  1. (D) The cardiac accelerator fibers originate in the T1-T4 segments. A high spinal, above T1, can cause bradycardia
    by anesthetizing these fibers. Diabetic patients who display orthostatic hypotension have an autonomic neuropathy.
    The cardiac accelerator fibers are essentially ablated in these patients and therefore, the slowing of heart
    rate does not ordinarily develop with high spinals. Pilocarpine, a parasympathomimetic agent, will not prevent
    bradycardia with spinal anesthesia. Patients with Wolff-Parkinson-White syndrome will become bradycardic
    when the autonomic accelerator fibers are interrupted, as will patients with a spinal cord transection below T4.
    Recent myocardial infarction does not eliminate susceptibility to bradycardia with sympatholysis unless the
    patient has a complete heart block (Hines: Stoelting’s Anesthesia and Co-Existing Disease, ed 5, pp 72, 242-243,
    374-375).
69
Q
  1. A 35-year-old woman receives a popliteal block for ankle and foot surgery. Which other nerve must be blocked in
    order to have complete anesthesia of the foot?
    A. Deep peroneal nerve
    B. Superficial peroneal nerve
    C. Sural nerve
    D. Saphenous nerve
    E. Posterior tibial nerve
A
  1. (D) All of the nerves of the foot with the exception of the saphenous are derived from the sciatic nerve. The sciatic
    nerve distally becomes the tibial and peroneal nerves which can be blocked at the popliteal fossa for surgery
    below the knee. The saphenous nerve is a branch of the femoral nerve and provides sensory innervation along
    the medial aspect of the lower leg between the knee and the medial malleolus and must also be blocked for surgery
    below the knee (Brown: Atlas of Regional Anesthesia, ed 3, pp 135-138, 142; Morgan: Clinical Anesthesiology,
    ed 4, pp 350-352).
70
Q
857. The most common complication of a celiac plexus block is
A. Hypotension
B. Seizure
C. Subarachnoid injection
D. Retroperitoneal hematoma
E. Constipation
A
  1. (A) The sympathectomy produced by a celiac plexus block causes hypotension by decreasing preload to the heart.
    This complication can be avoided by volume loading the patient with lactated Ringer’s solution. Subarachnoid
    injection is the most serious complication of celiac plexus block. Seizure is possible with an intravascular injection.
    Retroperitoneal hematoma is also possible, but extremely rare. This block frequently relieves constipation
    by interrupting the sympathetic fibers and leaving the parasympathetic fibers unopposed (Barash: Clinical Anesthesia,
    ed 5, pp 737-738).
71
Q
  1. The occipital portion of the skull receives sensory innervation from
    A. Spinal accessory nerve (nerve XI)
    B. Facial nerve (nerve VII)
    C. Ophthalmic branch of trigeminal nerve (nerve V)
    D. Maxillary branch of trigeminal nerve (nerve V)
    E. None of the above
A
  1. (E) The occiput receives sensory innervation from the greater and lesser occipital nerves, which are terminal branches
    of the cervical plexus. Blockade of these nerves is usually carried out as a diagnostic step in the evaluation of head
    and neck pain (Barash: Clinical Anesthesia, ed 5, p 724; Brown: Atlas of Regional Anesthesia, ed 3, p 155).
72
Q
859. Each of the following is a potential complication of lumbar sympathetic blocks EXCEPT
A. Puncture of the renal pelvis
B. Intravascular injection (aorta)
C. Seizure
D. S1 nerve block
E. Accidental subarachnoid injection
A
  1. (D) Potential complications from lumbar sympathetic block include subarachnoid injection, puncture of a
    major vessel (e.g., aorta) or renal pelvis, neuralgia, somatic nerve damage, perforation of a disk, infection,
    ejaculatory failure, and chronic back pain. Blockade of nerves arising from the lumbar plexus is possible,
    but given the anatomic location of the sacral plexus, blockade of an S1 nerve would be extremely unlikely
    if not impossible (Brown: Atlas of Regional Anesthesia, ed 3, p 309; Raj: Practical Management of Pain,
    ed 3, p 678).
73
Q
  1. After placement of an epidural catheter in a 55-year-old patient for total hip arthroplasty, an entire epidural dose is
    administered into the subarachnoid space. Physiologic effects consistent with subarachnoid injection of large volumes
    of local anesthetic include all of the following EXCEPT
    A. Hypotension
    B. Bradycardia
    C. Respiratory depression
    D. Constricted pupils
    E. Possible cauda equina syndrome
A
  1. (D) With the unintentional injection of an epidural dose of local anesthetic into the subarachnoid space, spinal
    anesthesia develops rapidly. Blockade of the sympathetic fibers (T1-L2) produces hypotension, particularly
    if the patient is hypovolemic. Bradycardia is produced by blocking the cardiac accelerator fibers (T1-T4).
    Respiratory arrest is due to hypoperfusion of the respiratory centers as well as paralysis of the phrenic nerve
    (C3-C5). The pupils become dilated after intrathecal injection of large quantities of local anesthetics; they
    will return to normal size after the block recedes. Cauda equina syndrome has occasionally developed when
    the epidural dose was unintentionally administered into the subarachnoid space (most commonly with chloroprocaine).
    If one suspects an unintentional placement of the epidural dose subarachnoid, supportive methods
    are initially done (the basic ABC’s of resuscitation). One can also aspirate CSF from the epidural catheter
    (if it was inserted) to help remove some of the drug as well as reducing the pressure in the subarachnoid
    space, which might help better perfuse the spinal cord and decrease the chance of cauda equina syndrome
    developing (Barash: Clinical Anesthesia, ed 5, p 712; Miller: Anesthesia, ed 6, pp 1660, 1676; Southorn: Reducing
    the potential morbidity of an unintentional spinal anaesthetic by aspirating cerebrospinal fluid, Br J Anaesth
    76:467-469, 1996).
74
Q
  1. A 49-year-old type I diabetic patient with a long history of burning pain in the right lower extremity receives a spinal
    anesthetic with 100 mg of procaine with 5% dextrose. The patient reports no relief in symptoms but has complete
    bilateral motor blockade. What diagnosis is consistent with this differential blockade examination?
    A. Diabetic neuropathy
    B. Central pain
    C. Myofascial pain
    D. Meralgia paresthetica
    E. Complex regional pain syndrome I (reflex sympathetic dystrophy)
A
  1. (B) Somatic pain in the extremities is relieved with spinal anesthesia. If a patient fails to obtain pain relief despite
    complete sympathetic, sensory, and motor blockade, a “central” mechanism for the pain is likely or the lesion
    causing the pain is higher in the CNS than the level of blockade achieved by the spinal. Central pain states may
    include encephalization, psychogenic pain, or malingering. Persistence of pain in the lower extremities after
    successful spinal blockade suggests a central source or psychological source of pain (Miller: Anesthesia, ed 6,
    pp 2763-2772).
75
Q
  1. An 18-year-old man has a seizure during placement of an interscalene brachial plexus block with 2% lidocaine. The
    anesthesiologist begins to hyperventilate the patient’s lungs with 100% O2 using an anesthesia bag and mask. The
    rationale for this therapy includes all of the following EXCEPT
    A. Helps to prevent and treat hypoxia
    B. Hyperventilation decreases blood flow and delivery of lidocaine to the brain
    C. Hyperventilation induces hypokalemia which elevates the seizure threshold
    D. Hyperventilation induces alkalosis which elevates the seizure threshold
    E. Hyperventilation induces alkalosis which converts lidocaine to the protonated (ionized) form
A
  1. (E) During a seizure, administration of 100% O2 helps to prevent and treat hypoxia in a patient who otherwise might
    not be breathing. Hyperventilation also causes cerebral vasoconstriction and decreased delivery of local anesthetic
    to the brain. Hyperventilation induces hypokalemia and respiratory alkalosis, both of which result in hyperpolarization
    of nerve membranes and elevation of the seizure threshold. Hyperventilation also raises the patient’s
    pH (respiratory alkalosis) and converts lidocaine into the non-ionized (nonprotonated) form, which crosses the
    membrane more easily than the ionized form, which is detrimental (Stoelting: Basics of Anesthesia, ed 5, p 130).
76
Q
863. Para-aminobenzoic acid is a metabolite of
A. Mepivacaine
B. Ropivacaine
C. Bupivacaine
D. Procaine
E. Prilocaine
A
  1. (D) Para-aminobenzoic acid is a metabolite of the ester-type local anesthetics. Local anesthetics may be placed into
    two distinct categories based on their chemical structure: ester or amide. The amides, which are ropivacaine
    lidocaine, etidocaine, prilocaine, mepivacaine and bupivacaine, are metabolized in the liver. The ester local anesthetics are cocaine, procaine, chloroprocaine, tetracaine, and benzocaine. These drugs are metabolized by
    the enzyme pseudocholinesterase found in the blood. Para-aminobenzoic acid is a metabolic breakdown product
    of ester anesthetic and is responsible for allergic reactions in some individuals (Stoelting: Pharmacology and
    Physiology in Anesthetic Practice, ed 4, pp 180-189).
77
Q
  1. Which statement concerning peripheral nerve structure and function is FALSE?
    A. Both nonmyelinated and myelinated nerves are surrounded by Schwann cells
    B. The speed of propagation of an action potential along a nerve axon is greatly enhanced by myelin
    C. Generation of an action potential is an “all-or-nothing” phenomenon
    D. Propagation of an action potential along myelinated nerve axons occurs by saltatory conduction via the nodes of
    Ranvier
    E. Myelination renders nerves less sensitive to local anesthetic blockade
A
  1. (E) Peripheral nerve axons are always enveloped by a Schwann cell. The myelinated nerves may be enveloped many
    times by the same Schwann cell. Transmission of nerve impulses (i.e., action potentials) along nonmyelinated
    nerves occurs in a continuous fashion, whereas transmission along myelinated nerves occurs by saltatory conduction
    from one node of Ranvier to the next. Myelination speeds transmission of neurological impulses; it also
    renders nerves more susceptible to local anesthetic blockade. An action potential is associated with an inward
    flux of sodium that occurs after a certain membrane threshold has been exceeded (Miller: Anesthesia, ed 6,
    pp 576-579).
78
Q
  1. A 42-year-old woman with a morbid fear of general anesthesia receives an interscalene block for shoulder arthroscopy
    consisting of 20 mL 0.5% ropivacaine. Much of her arm, shoulder and hand are numb, but the patient complains of
    pain as the incision is made at the upper portion of the shoulder. The most appropriate next step is
    A. Repeat block
    B. Perform intercostobrachial block
    C. Perform superficial cervical plexus block
    D. Perform a deep cervical plexus block
    E. Induce general anesthesia
A
  1. (C) The needle insertion site for an interscalene block is C6. Local anesthetics usually spread to C5, C6 and
    C7 which supply much, but not all, of the cutaneous innervation to the shoulder. With low-to-moderate
    volume blocks there will be sparing of the (C3-C4) nerve roots, which supply some of the innervation to
    the anterior shoulder. Of note, C8 and T1 may also be spared, often resulting in the need for ulnar nerve
    supplementation if this block were used for a hand operation. Complete anesthesia for shoulder arthroscopy
    may require a supplemental superficial cervical plexus with use of low to moderate volumes of a local
    anesthetic (Hebl: Mayo Clinic Atlas of Regional Anesthesia and Ultrasound Guided Peripheral Nerve Blockade,
    Chapter 9, in press).
79
Q
  1. According to the 2004 American Society of Regional Anesthesia and Pain Medicine (ASRA) practice advisory on
    infectious complications of regional anesthesia and pain medicine, the most important action to maintain aseptic
    technique and prevent cross-contamination during regional anesthesia techniques is
    A. Wearing surgical gown
    B. Hand washing
    C. Using soap and water instead of alcohol-based antiseptics
    D. Keeping fingernails short
    E. Using povidone iodine (e.g., Betadine) instead of alcohol-based chlorhexidine to scrub
A
  1. (B) Hand washing is one of the most important techniques to prevent infections especially when alcohol-based antiseptic
    solutions are used with sterile gloves. Although soap and water remove bacteria, they do not effectively kill
    organisms. Antiseptic solutions with alcohol appear better than nonalcoholic antiseptics (e.g., povidone iodine).
    Nail length does not appear to be a risk factor for infections, because the majority of bacterial growth occurs
    along the proximal 1 mm of nail adjacent to the subungual skin. Universal use of gowns and gloves does not
    appear to be better than gloves alone in preventing infections in ICUs and presumably is less important than
    adequate hand washing and use of sterile gloves (Hebl JR: Infections complications: A new practice advisory - The
    importance and implications of aseptic techniques during regional anesthesia. Reg Anesth Pain Med, 31:289-290,
    311-323, 2006).
80
Q
  1. A 75-year-old woman with a history of pulmonary embolism is scheduled for a right lower lobectomy for lung cancer.
    She is receiving dalteparin (Fragmin) for deep vein thrombosis (DVT) prophylaxis. How long after her last dose
    should one wait prior to placement of a thoracic epidural?
    A. 12 hours
    B. 24 hours
    C. 30 hours
    D. 72 hours
    E. No waiting necessary
A
  1. (A) In patients taking low-molecular weight heparin, or LMWH (e.g., enoxaparin, dalteparin, tinzaparin), caution
    should be exercised before proceeding with an epidural or spinal anesthetic because of the risk of producing
    an epidural or spinal hematoma. The amount of time between the last dose of the LMWH and the relative
    safety of starting a central neuraxial block depends on the dose of the LMWH. At the lower doses, used for
    thromboprophylaxis, the LMWH should be held at least 10 to 12 hours prior to the block. At the higher doses,
    used to treat an established DVT, one should wait at least 24 hours after the last dose of LMWH prior to
    the block (Barash: Clinical Anesthesia, ed 5, p 713; Miller: Anesthesia, ed 6, pp 1677, 2742-2743; Second Consensus
    Conference on Neuraxial Anesthesia and Anticoagulation, April 25-28, 2002, www.asra.com/consensusstatements/
    2.html).
81
Q
868. How long should a patient be off clopidogrel (Plavix) before performing a central neuraxial block?
A. 1 day
B. 2 days
C. 7 days
D. 14 days
E. No waiting necessary
A
  1. (C) Patients taking nonsteroidal anti-inflammatory drugs (NSAIDs), ticlopidine and clopidogrel, exert effects on
    platelet function. Nonsteroidal anti-inflammatory drugs (NSAIDs) are not a problem if given alone before epidural
    or spinal anesthesia. But patients taking ticlopidine should wait 14 days and patients taking clopidogrel
    should wait 7 days before having a neuraxial block placed, because of the increased risk of spinal hematoma
    formation. Keep in mind that caution is always needed and the ASRA statement of “Careful preoperative assessment
    of the patient to identify alterations of health that might contribute to bleeding is crucial” is important
    (Barash: Clinical Anesthesia, ed 5, p 713; Second Consensus Conference on Neuraxial Anesthesia and Anticoagulation,
    April 25-28, 2002, www.asra.com/consensus-statements/2.html).
82
Q
869. Addition of bicarbonate to local anesthetics results in
A. Delayed onset of action
B. Reduced toxicity
C. Increased duration of action
D. Increased anesthetic potency
E. Reduced pain with skin infiltration
A
  1. (E) Adding sodium bicarbonate to local anesthetic solutions hastens the onset of action of the local anesthetics,
    especially when the local anesthetic solution contains epinephrine (which is produced at a lower pH). By raising
    the pH, more of the local anesthetic is in the non-ionized, more lipid-soluble state. Raising the pH too
    much (i.e., >6.05-8) would cause precipitation of the local anesthetic. It also seems to decrease pain with skin
    infiltration. Pain on injection can also be decreased by a slow injection of the local anesthetic (Barash: Clinical
    Anesthesia, ed 5, p 460; Miller: Anesthesia, ed 6, p 585).
83
Q
870. Through which of the following would a spinal needle NOT pass during a midline placement of a subarachnoid block
in the L3-L4 lumbar space?
A. Supraspinous ligament
B. Interspinous ligament
C. Ligamentum flavum
D. Posterior longitudinal ligament
E. Dura mater
A
  1. (D)
    This figure shows the anatomic structures that must be traversed by the spinal needle during performance of a
    subarachnoid block. The structures include the skin, subcutaneous tissue, supraspinous ligament, interspinous
    ligament, the ligamentum flavum, and finally the dura (posterior). If you were to continue to advance the spinal
    needle, you would encounter the dura (anteriorly) while exiting the subarachnoid space, the posterior longitudinal
    ligament, the periosteum of the vertebral body, and finally, bone (Cousins: Neural Blockade in Clinical
    Anesthesia and Management of Pain, ed 3, p 205).
84
Q
871. What epidural dose of bupivacaine will give similar sensory analgesia as 10 mL of 2% lidocaine?
A. 5 mL of 0.25%
B. 10 mL of 0.25%
C. 5 mL of 0.5%
D. 10 mL of 0.5%
E. 5 mL of 0.75%
A
  1. (D) In the epidural space, bupivacaine is four times more potent than lidocaine, so 0.5% bupivacaine is similar to
    2% lidocaine. The duration of the bupivacaine block will be longer because bupivacaine has a long duration of
    action and lidocaine has an intermediate duration of action (Barash: Clinical Anesthesia, ed 5, pp 459-463).
85
Q
  1. Each of the following additives to a spinal anesthetic possesses analgesic properties EXCEPT
    A. Clonidine
    B. Neostigmine
    C. Epinephrine
    D. Fentanyl
    E. All of the above posses analgesic activity
A
  1. (E) Drugs with α-agonist activity (phenylephrine/epinephrine) possess some analgesic activity but less than narcotics
    and local anesthetics. In addition, intrathecal epinephrine will reduce systemic/vascular uptake of local
    anesthetics, thereby enhancing their effects, including hypotension. Clonidine alone, when administered neuraxially,
    is an effective analgesic. Neostigmine has some mild analgesia properties but experience is limited.
    Opioids added to the spinal solution enhance surgical anesthesia and provide postoperative pain. Fentanyl
    25 μg is commonly added for short surgical procedures (outpatient) whereas morphine can be used when longer
    postoperative analgesia is desired for inpatients (Morgan: Clinical Anesthesiology, ed 4, p 309; Stoelting: Basics of
    Anesthesia, ed 5, p 256).
86
Q
873. Which of the following local anesthetics is inappropriately paired with a clinical application because of its properties
or toxicity?
A. Tetracaine, topical anesthesia
B. Bupivacaine, intravenous anesthesia
C. Prilocaine, infiltrative anesthesia
D. Chloroprocaine, epidural anesthesia
E. Ropivacaine, epidural anesthesia
A
  1. (B) For topical anesthesia, lidocaine, tetracaine, dibucaine and benzocaine are effective, as well as the combination
    of lidocaine and prilocaine or EMLA cream. For intravenous regional anesthesia or Bier blocks, many drugs
    have been used. Ester local anesthetics are not used for IV regional blocks because they can be broken down in
    the blood stream (by plasma ester hydrolysis) which can shorten the drug’s duration of action and can also cause
    thrombophlebitis of the vein (reported with chloroprocaine). Because cardiovascular collapse has been reported
    with bupivacaine, and would likely also occur with etidocaine and ropivacaine, if the tourniquet unintentionally
    is released while the block is setting up, they are not used for intravenous regional anesthesia. Lidocaine and
    prilocaine are used for Bier blocks because of their relative safety. For infiltrative anesthesia, all local anesthetics
    can be used. All local anesthetics can be used in the epidural space, although procaine and tetracaine are rarely
    used (procaine has a slow onset and tetracaine has marked motor block) (Miller: Anesthesia, ed 6, pp 586-591;
    Stoelting: Basics of Anesthesia, ed 5, p 127).
87
Q
  1. Discharge criteria from the post-anesthesia care unit (PACU) would be reached fastest after a 20 to 30 mL volume of
    which of the following epidurally administered local anesthetics?
    A. 3% 2-chloroprocaine
    B. 2% lidocaine
    C. 1% etidocaine
    D. 0.75% ropivacaine
    E. 0.5% levobupivacaine
A
  1. (A) Procaine and 2-chloroprocaine have a short duration of action; lidocaine, mepivacaine and prilocaine have an
    intermediate duration of action; etidocaine, bupivacaine, levobupivacaine, tetracaine and ropivacaine have a
    long duration of action. For similar sensory anesthesia, a higher concentration of local anesthetic is needed for
    the short duration of local anesthetics compared with both the intermediate and long duration agents, because
    they are less potent (Barash: Clinical Anesthesia, ed 5, pp 705-707; Miller: Anesthesia, ed 6, pp 1674-1675).
88
Q
  1. A caudal block with 0.25% bupivacaine and 1:200,000 epinephrine is planned for postoperative analgesia after bilateral
    inguinal hernia repair in a 5 month old. Each of the following would be consistent with an intravascular injection
    EXCEPT
    A. Systolic blood pressure increase by 15 mm Hg
    B. Heart rate decrease by 10 bpm
    C. Ventricular extrasystoles
    D. Seizure
    E. T-wave amplitude of 25 percent over baseline
A
  1. (B) A change in the T-wave amplitude of 25 percent, an increase in heart rate of 10 beats per minutes, or systolic blood
    pressure greater than 15 mm Hg is considered a positive response to an epinephrine containing local anesthetic
    solution. A slight drop in heart rate may result if the block is properly performed and no intravascular injection
    occurs (Motoyama: Smith’s Anesthesia for Infants and Children, ed 7, p 464).
89
Q
876. Which is NOT a potential complication of a stellate ganglion block
A. Recurrent laryngeal nerve paralysis
B. Subarachnoid block
C. Brachial plexus block
D. Pneumothorax
E. Increased heart rate
A
  1. (E) All of the choices listed are potential complications of stellate ganglion blockade except an increase in heart rate.
    The stellate ganglion supplies sympathetic fibers to the upper extremity and head and some to the heart. Loss of
    the cardiac acceleratory fibers may slow the heart rate, not speed it up. Other potential complications of stellate
    ganglion blockade include accidental injection of the local anesthetic into a vertebral artery resulting in seizure
    and inadvertent cervical epidural (Barash: Clinical Anesthesia, ed 5, pp 736-737).
90
Q
  1. An axillary block utilizing the transarterial approach with 0.5% bupivacaine and epinephrine (1:200,000) is performed
    in a 70-kg patient. Thirty mL is injected posterior to the axillary artery and 30 mL anterior to it. How many
    mg have been injected and was the maximum recommended dose exceeded?
    A. 150 mg bupivacaine, 150 μg epinephrine did not exceed maximum dose
    B. 150 mg bupivacaine, 150 μg epinephrine exceeded maximum dose
    C. 300 mg bupivacaine, 300 μg epinephrine did not exceed maximum dose
    D. 300 mg bupivacaine, 300 μg epinephrine exceeded maximum dose
    E. Transarterial blocks should never contain epinephrine and the block should not be done
A
  1. (D) A total of 60 mL of 0.5% bupivacaine with epinephrine (1:200,000) was used. A 0.5% solution = 0.5 g
    in 100 mL of fluid = 500 mg/100 mL = 5 mg/mL. A 1:200,000 solution means 1 gram in 200,000 mL =
    1000 mg/200,000 mL = 1 mg/200 mL = 1000 μg/200 mL = 5 μg/mL. Therefore 60 mL of 0.5% bupivacaine
    with 1:200,000 epinephrine contains 60 mL × 5 mg/mL or 300 mg bupivacaine and 60 mL × 5 μg/mL or 300 μg
    of epinephrine. For a major nerve block the maximum recommended dose with epinephrine (1:200,000) is
    500 mg for lidocaine and mepivacaine, 600 mg with prilocaine, and 225 mg with bupivacaine. Epinephrine is
    used in the local anesthetic to check for intravascular injection of the incremental doses and is not contraindicated
    but should be used for this block (Miller: Anesthesia, ed 6, p 588).
91
Q
878. Five days after knee arthroscopy under spinal anesthesia, a 55 year old complains of double vision and difficulty hearing.
The other likely finding would be
A. Headache
B. Fever
C. Weakness in legs
D. Mental status changes
E. Backache
A
  1. (A) Post dural puncture headaches (spinal headaches) usually develop within 12 to 48 hours after a dural puncture,
    but may develop immediately or take months to develop. The most characteristic symptom is a postural component
    where the headache occurs in the upright position and is usually completely gone when the patient is
    in the supine position. The headache is typically frontal and/or occipital in location. Other symptoms include
    nausea, vomiting, anorexia, visual disturbances (blurred vision, double vision, photophobia) and occasionally
    hearing loss (routinely found with auditory testing) (Stoelting: Basics of Anesthesia, ed 5, p 878).
92
Q
879. Administration of an interscalene block is associated virtually 100% of the time with
A. Hoarseness
B. Ulnar nerve blockade
C. Ipsilateral Horner’s syndrome
D. Diaphragmatic hemiparalysis
E. Bradycardia
A
  1. (D) Ipsilateral phrenic nerve block with diaphragmatic paralysis occurs is virtually 100% of patients receiving an
    interscalene block. This produces a 25% reduction in pulmonary function, making this block a contraindication
    in patients with borderline pulmonary function. Blockage of the recurrent laryngeal nerve can occur but
    is rare; however, if the patient has contralateral vocal cord palsy and develops a recurrent laryngeal nerve block,
    complete airway obstruction can occur. With this block, the inferior trunk of the brachial plexus where the ulnar
    nerve is derived may be spared (Miller: Anesthesia, ed 6, p 1688; Stoelting: Basics of Anesthesia, ed 5, pp 276-278).
93
Q
880. Which of the following nerves can be electrically stimulated at the ankle to produce flexion of the toes?
A. Posterior tibial nerve
B. Saphenous nerve
C. Deep peroneal nerve
D. Superficial peroneal nerve
E. Sural nerve
A
  1. (A) Five nerves are blocked when performing an ankle block. The saphenous, superficial peroneal, and sural nerves
    are all sensory below the ankle and electrical stimulation would have no effect. Stimulation of the posterior
    tibial nerve causes flexion of the toes by stimulating the flexor digitorum brevis muscles and abduction of the
    first toe by stimulating the abductor hallucis muscles. The posterior tibial nerve also is sensory to most of the
    plantar part of the foot. Stimulation of the deep peroneal nerve causes extension of the toes by stimulating the
    extensor digitorum brevis muscles. The deep peroneal nerve has a small sensory branch for the first interdigital
    cleft. From the practical standpoint, many anesthesiologists perform a purely infiltration block of these nerves.
    If a nerve stimulator is used, it is mainly used to find the posterior tibial nerve, which can be hard to anesthetize
    if small volumes of local anesthetic are administered. The posterior tibial nerve can be difficult to stimulate in
    diabetics with diabetic neuropathy (Loeser: Bonica’s Management of Pain, ed 3, pp 1614-1627).
94
Q
  1. Which of the following observations, after nerve injury, is correctly paired with the appropriate nerve?
    A. Inability to flex the forearm—ulnar nerve
    B. Numbness in the thumb—radial nerve
    C. Inability to extend the forearm—musculocutaneous nerve
    D. Numbness in the little finger—median nerve
    E. All are correctly paired
A
  1. (B) The motor responses include: arm flexion at the elbow (musculocutaneous nerve), arm extension at the elbow
    (radial nerve), forearm pronation, wrist flexion and thumb opposition (median nerve), ulnar deviation of the
    wrist, little finger flexion, thumb adduction and flaring of the fingers (ulnar nerve), wrist and finger extension
    (radial nerve). The sensory response (includes some variations) is: back of the arm, forearm and radial side dorsal
    side of the hand (radial nerve), skin of the lateral forearm (musculocutaneous nerve), ulnar side of the hand and
    both surfaces of the ulnar one and one-half fingers (ulnar nerve), the radial side of the palm of the hand as well
    as the dorsal aspect of the radial three and one-half fingers (median nerve). To evaluate the setup of a brachial
    plexus block, a common technique is to perform the four P’s (Push, Pull, Pinch, Pinch). Have the patient push
    or extend the forearm (radial), pull or flex the forearm (musculocutaneous nerve), pinch the index or second
    finger (median nerve), pinch the little finger (ulnar nerve) (Brown: Atlas of Regional Anesthesia, ed 3, pp 27-35;
    Neal JM, et al. Upper Extremity Regional Anesthesia - Essentials of Our Current Understanding, 2008. Reg Anesth
    Pain Med 34:134-170, 2009).
95
Q
  1. During an airway exam, a 53-year-old patient mentions that his right thumb tingles and then becomes numb if he
    extends his head for more than a few seconds. This symptom most likely represents a (an)
    A. Unstable c-spine
    B. Lhermitte’s phenomenon
    C. C6 nerve root irritation
    D. C8 radiculopathy
    E. Carpal tunnel syndrome
A
  1. (C) Unilateral numbness or paresthesia in the upper extremity during extension of the neck usually represents nerve
    root impingement at the vertebral foramina. Specifically, unilateral degenerative changes restrict the foramen to
    such a degree that it compresses and irritates the nerve root traversing the vertebral foramen when the head is
    extended. Treatment ranges from NSAIDs, to steroids and may require surgical intervention if there is muscle
    weakness (Stoelting: Basics of Anesthesia, ed 5, p 249; Miller: Anesthesia, ed 6, p 1687).
96
Q
  1. When performing an interscalene block with a peripheral nerve stimulator you note diaphragmatic movement. You
    should now
    A. Inject the local anesthetic as the needle is in an appropriate location
    B. Redirect the needle in an anterior direction
    C. Redirect the needle in a more cephalad direction
    D. Redirect the needle in a posterior direction
    E. Advance the needle about 0.5 cm more and inject
A
  1. (D) Although a successful interscalene block causes ipsilateral phrenic nerve paralysis in almost 100% of patients,
    identifying the phrenic nerve means you are anterior to the brachial plexus and you should reposition your
    needle. You should redirect the needle in a posterior direction (Barash: Clinical Anesthesia, ed 5, pp 726-728;
    Miller: Anesthesia, ed 6, pp 1686-1689).
97
Q
884. During placement of an interscalene block the patient becomes hypotensive, bradycardic and cyanotic. Most likely
cause is
A. Vertebral artery injection
B. Carotid artery injection
C. Phrenic nerve blockade
D. Total spinal
E. Stellate ganglion block
A
  1. (D) With an intravascular injection, the main symptoms would most likely be CNS toxicity (e.g., seizures) as
    blood flow is directly to the brain. The Bezold-Jarish reflex (hypotension and bradycardia) has been reported
    in awake, sitting patients undergoing shoulder surgery with an interscalene block. This maybe related to intracardiac
    mechanoreceptors being stimulated by the decreased venous return in the sitting position. This leads to
    decreased sympathetic tone and increased parasympathetic tone. Breathing is still present with this reflex. Block
    of the stellate ganglion would produce a Horner’s syndrome, which is not associated with breathing abnormalities.
    Injection into the intrathecal space is uncommon, but possible, especially if the needle is not pointed in the
    caudal direction, and would lead to a total spinal block with little local anesthetic injected (e.g., hypotension,
    bradycardia respiratory paralysis that would lead to cyanosis) (Barash: Clinical Anesthesia, ed 5, pp 726-728;
    Miller: Anesthesia, ed 6, pp 1686-1689).
98
Q
885. The reason that ropivacaine is marketed as pure S enantiomers is because the S form is associated with
A. More rapid onset
B. Longer duration
C. Reduced cardiac toxicity
D. Reduced incidence of anaphylaxis
E. Less motor blockade
A
  1. (C) The pipecoloxylidide local anesthetics (mepivacaine, bupivacaine, ropivacaine and levobupivacaine) are chiral
    drugs, which means they have an asymmetric carbon atom (i.e., have a left- or S and a right- or R hand configuration).
    Mepivacaine and bupivacaine are racemic mixtures (50% S: 50% R mixture). The pure S forms show
    reduced neurotoxicity and reduced cardiotoxicity (e.g., ropivacaine and levobupivacaine). Lidocaine is an achiral
    compound (i.e., has no chiral carbon atom) (Barash: Clinical Anesthesia, ed 5, p 458; Stoelting: Pharmacology
    and Physiology in Anesthetic Practice, ed 4, pp 180-181).
99
Q
886. Nerves that originate from the sacral plexus include each of the following EXCEPT
A. Femoral nerve
B. Tibial nerve
C. Superficial peroneal nerve
D. Deep peroneal nerve
E. Sural nerve
A
  1. (A) Nerves to the lower extremity emerge from the L2 to S3 nerve roots. The upper roots (mainly L2-L4) form the
    lumbar plexus which gives rise to the femoral, obturator, and lateral femoral cutaneous nerves. A branch from
    the lumbar plexus along with the sacral plexus gives rise to the sciatic nerve. Branches of the sciatic nerve include
    the common peroneal (branches to make the superficial and deep) and the tibial, and the sural nerves (Barash:
    Clinical Anesthesia, ed 5, pp 739-740; Miller: Anesthesia, ed 6, pp 1695-1696).
100
Q
887. The only technique shown to prevent anesthetic-related nerve injury (ARNI) during placement of peripheral nerve
blocks is
A. Ultrasound guided regional technique
B. Transarterial technique
C. Nerve stimulator
D. Paresthesia technique
E. None of the above
A
  1. (E) Anesthetic-related nerve injuries to the brachial plexus are rare and poorly understood. The only way to minimize
    nerve injury is to minimize trauma to neural fibers. Although ultrasound-guided technique is promising,
    currently there is no clinical evidence for this (Neal JM, et al. Upper extremity regional anesthesia - Essentials of
    our current understanding, 2008. Reg Anesth Pain Med, 34:134-170, 2009).
101
Q
  1. An axillary block is performed on a healthy 19-year-old athlete. Thirty mL of 0.75% bupivacaine is injected incrementally.
    Fifteen minutes after the bupivacaine injection, the patient has a seizure and experiences a ventricular fibrillation
    arrest. Which of the measures below is NOT indicated.
    A. Begin chest compressions at 100 per minute
    B. Ventilate with 100% oxygen
    C. Bolus propofol to bind local anesthetic
    D. Infuse 20% lipid emulsion
    E. Repeat emulsion infusion if fibrillation continues
A
  1. (C) Local anesthetic toxicity is a multisystem phenomenon, but the most crucial manifestation involves the heart
    (atrioventricular conduction block, arrhythmias, myocardial depression, and cardiac arrest). In isolated cardiac
    tissue hypercarbia, acidosis and hypoxia will augment the negative inotropic and chronotropic effects of bupivacaine.
    In the event of seizure and ventricular fibrillation, hypercarbia and hypotension would cause a severe
    acidosis and hypoxia, which would potentiate the toxicity of bupivacaine. To reduce these cardiotoxic effects,
    the initial resuscitation efforts are aimed at maximizing oxygen delivery, increasing tissue perfusion and ventilation.
    This is accomplished through chest compressions, assisted ventilation with oxygen, intravenous sodium
    bicarbonate, and inotropic and chronotropic support with pharmacological agents such as atropine, epinephrine,
    dopamine, and dobutamine. Standard cardiopulmonary resuscitation may not be successful in some cases,
    and placement of the patient on cardiopulmonary bypass may be necessary. Recently, lipid infusions have been
    shown to reduce the toxicity of intravascularly injected bupivacaine. Propofol is formulated as a lipid emulsion,
    and as such would bind bupivacaine to some degree, but this effect would be overshadowed by the substantial
    cardiac depressant of the anesthetic (Miller: Anesthesia, ed 5, pp 592-596, 1731-1732).
102
Q
889. The structure LEAST likely to be encountered during placement of interscalene block is
A. Phrenic nerve
B. Vertebral artery
C. Recurrent laryngeal nerve
D. Subarachnoid space
E. Vagus nerve
A
  1. (E) When performing an interscalene block, the needle is usually placed at the line extending lateral to the cricoid
    cartilage that intersects the interscalene groove at the C6 level. The needle in inserted in a slightly posterior and
    a 45° caudad direction. The caudad direction is used to decrease the chance of injecting the local anesthetic into the vertebral artery, or obtaining a spinal or epidural block. The phrenic nerve is routinely blocked (100% of the
    time) and occasionally the recurrent laryngeal nerve is blocked (Stoelting: Basics of Anesthesia, ed 5, pp 276-278;
    Miller: Anesthesia, ed 6, pp 1686-1689).
103
Q
890. All of the following are symptoms of a developing epidural hematoma EXCEPT
A. Radicular back pain
B. Bowel and bladder dysfunction
C. Sensory deficits
D. Motor deficits
E. Fever and chills
A
  1. (E) Epidural hematomas are rare complications of spinal anesthesia (1:200,000) and epidural anesthesia (1:150,000).
    However, in the presence of LMWH the incidence is much higher: 1:40,000 with spinal anesthesia and 1:3000
    with continuous epidural catheter. Clinical symptoms include radicular back pain, bowel and bladder dysfunction,
    sensory or motor deficits. An MRI is the diagnostic test of choice and prompt decompressive laminectomy
    is the treatment of choice (Fleischer: Anesthesia and Uncommon Diseases, ed 5, p 573).
104
Q
891. In addition to C nerve fibers, which nerve fibers carry pain impulses?
A. A-α
B. A-β
C. A-γ
D. A-δ
E. B
A
  1. (D) Peripheral nerves are classified according to the fiber size and physiologic properties such as the presence or
    absence of myelin, conduction velocity, location, and function. Type A fibers range in diameter from 1 to 22
    μm, are myelinated, and have moderate-to-fast conduction velocities. These fibers are subclassified into four
    groups based on their location and function. Type A-alpha and A-beta fibers provide motor and proprioception
    function to muscles and joints; type A-gamma fibers innervate muscle spindles and provide for muscle tone;
    and type A-delta fibers provide pain, temperature and touch sensation. Type B fibers are preganglionic sympathetic
    nerves that are less than 3 μm in diameter, myelinated, and have medium conduction velocities. Type C
    fibers are postganglionic sympathetic nerves that are very small in diameter, are not myelinated, and have slow
    conduction velocities. Type C fibers are also afferent sensory nerves involved in pain, temperature, and touch
    (Miller: Anesthesia, ed 6, p 577).
105
Q
  1. An intradural mass lesion at the tip of a drug infusion catheter is LEAST likely to present as
    A. Piloerection, rhinorrhea, sweating, and more pain
    B. Development of numbness in T8 dermatomal pattern
    C. Hypopnea
    D. Perianal numbness
    E. Unilateral hip flexor weakness
A
  1. (C) Overdose of intrathecal opiates would not be a sign of an intradural mass lesion. Granulomas at the tip of
    intrathecal catheters used with intrathecal drug delivery systems are gaining increased attention. Granulomas are
    more frequently associated with high concentrations and doses of either morphine (>10 mg/day) or hydromorphone
    (>10 mg/day). Most patients who will develop granulomas receive the intrathecal medications for more
    than 6 months. Presenting symptoms may include loss of drug effect, new pain or paresthesias or neurologic
    deficits. Patients should be routinely screened for signs and symptoms of granuloma formation at scheduled
    intrathecal pump refill appointments. Suspicious cases should undergo prompt diagnostic imaging and consideration
    of neurosurgical consultation (Barash: Clinical Anesthesia, ed 5, p 1468).
106
Q
  1. Benzocaine is unique among the local anesthetics for which of the following reasons?
    A. It is a weak acid
    B. It can be used topically
    C. It is metabolized by same enzyme as succinylcholine
    D. It can promote formation of methemoglobin
    E. Is a vasoconstrictor
A
  1. (A). In addition to benzocaine, tetracaine and lidocaine can also be used as topical anesthetics. Pseudocholinesterase,
    the enzyme responsible for the metabolism of succinylcholine, metabolizes all of the ester local
    anesthetics, benzocaine, procaine, chloroprocaine and tetracaine. Benzocaine does promote the formation
    of methemoglobin, but is not alone in that regard since prilocaine also causes formation of methemoglobin.
    The pKa of benzocaine is 3.5, which qualifies it as a weak acid and as such exists in uncharged at physiologic
    pH. All other local anesthetic pKa’s are higher than 7.4, meaning that some fraction of them exists in the
    protonated form (Stoelting: Basics of Anesthesia, ed 5, p 127; Stoelting: Pharmacology and Physiology in Anesthetic
    Practice, ed 4, p 187).
107
Q
  1. Which statement concerning local anesthetics is CORRECT?
    A. The un-ionized form of a local anesthetic binds to the nerve membrane to actually block conduction
    B. If one node of Ranvier is blocked, conduction will be reliably interrupted
    C. The ability of a local anesthetic to block nerve conduction is directly proportional to the diameter of the fiber
    D. The presence of myelin enhances the ability of a local anesthetic to block nerve conduction
    E. Local anesthetics block transmission by inhibiting the voltage-gated potassium ion channels
A
  1. (D) The un-ionized form of the local anesthetic traverses the nerve membrane whereas the ionized form actually
    blocks conduction. About three nodes of Ranvier must be blocked to achieve anesthesia. The ability of a
    local anesthetic to block conduction is inversely proportional to the diameter of the fiber. The presence of
    myelin enhances the ability of a local anesthetic to block conduction, as does rapid firing. The local anesthetic
    blocks nerve transmission by inhibiting the voltage-gated sodium ion channels (Stoelting: Basics of
    Anesthesia, ed 5, pp 124-128).
108
Q
  1. Postdural puncture headaches
    A. Usually occur immediately following dural puncture
    B. Are relieved 8 to 12 hours after an epidural blood patch is performed
    C. Occur more frequently in nonpregnant patients compared with pregnant patients
    D. Can be associated with neurologic deficits
    E. Are more frequent in the elderly compared with younger adults
A
  1. (D) Postdural puncture headaches (PDPHs) typically appear within 12 to 48 hours of a dural puncture but may
    be immediate and occasionally have become delayed for several days or months after a dural puncture. The
    headaches are characterized by dull or throbbing frontal or occipital pain, which worsens with sitting and
    improves with reclining. Postspinal headaches may be associated with neurologic symptoms such as diplopia,
    tinnitus, and reduced hearing acuity. Very rarely, a subdural hematoma will develop. The etiology of postspinal
    headaches is believed to be caused by a reduction in CSF pressure and resulting tension on meningeal vessels
    and nerves (which results from leakage of CSF through the needle hole in the dura mater). Factors associated
    with an increased incidence of postspinal headaches include pregnancy, size and type of needle used to perform
    the block (larger needles and Quincke more common than smaller needles and Whitacre or Sprotte), and the
    number of dural punctures. They occur more frequently in young adults compared with children and the
    elderly. Conservative therapy for a postspinal headache includes bed rest, analgesics, and oral and intravenous hydration. If conservative therapy is not successful after 24 to 48 hours, an epidural “blood patch” with 10 to
    20 mL of the patient’s blood can be performed. An epidural “blood patch” usually provides prompt relief of the
    postspinal headache (Stoelting: Basics of Anesthesia, ed 5, pp 260-261).
109
Q
  1. Which of the following procedures for treatment of chronic pain requires localization of the epidural space with an
    epidural needle as part of technique?
    A. Transcutaneous electrical nerve stimulation
    B. Nucleoplasty
    C. Spinal cord stimulation
    D. Intradiskal electrothermal therapy
    E. Annuloplasty
A
  1. (C) When inserting a spinal cord stimulator, a 15 gauge needle is advanced into the epidural space via the
    paramedian approach. After confirmation of proper needle placement with AP and lateral fluoroscopic
    views, the stimulation electrode is passed through the needle and threaded to the desired vertebral level.
    The needle is then removed and the leads attached to the external programmer. Transcutaneous electrical
    nerve stimulation (TENS) unit electrodes are applied to the skin. With nucleoplasty, intradiskal electrothermal
    therapy and annuloplasty, the needle is inserted to access the intervertebral disk (Stoelting: Basics
    of Anesthesia, ed 5, pp 633-637).
110
Q
897. Each of the following drugs has been used to treat neuropathic pain. Selective inhibition of serotonin and norepinephrine
reuptake is the mechanism of which drug?
A. Duloxetine
B. Mexiletine
C. Gabapentin
D. Tramadol
E. Carbamazepine
A
  1. (A) Many drugs have been used to treat neuropathic pain, including analgesics (NSAIDs and opioids), first-generation
    antiepileptic drugs (e.g., carbamazepine and phenytoin), second-generation antiepileptic drugs (e.g.,
    gabapentin, pregabalin), topical agents (e.g., lidocaine, capsaicin), antiarrhythmics (e.g., mexiletine), tricyclic
    antidepressants (e.g., amitriptyline, nortriptyline, desipramine) as well as other antidepressants (e.g., duloxetine,
    venlafaxine). Duloxetine (Cymbalta) is a selective serotonin and norepinephrine reuptake inhibitor (SNRI) that
    is used for major depressive disorders, generalized anxiety disorders, fibromyalgia and diabetic peripheral neuropathic
    pain. Mexiletine is an orally effective amine analogue of lidocaine and may be effective in decreasing
    neuropathic pain when other drugs have failed. Gabapentin, a structural analogue of GABA, works by increasing
    the synthesis of the inhibitory neurotransmitter GABA. Tramadol is an opioid agonist that has moderate
    activity at the μ receptor and weak activity at the κ and δ opioid receptors. In addition, tramadol enhances the
    spinal inhibitory pathways by inhibiting the neuronal uptake of norepinephrine and serotonin as well as presynaptic
    stimulation of serotonin release. Carbamazepine (Tegretol) is an anticonvulsant with specific analgesic
    properties for trigeminal neuralgia. Carbamazepine seems to reduce polysynaptic responses by an unknown
    mechanism (Cousins: Neural Blockade in Clinical Anesthesia and Pain Medicine, ed 4, p 1065; Morgan: Clinical
    Anesthesiology, ed 4, pp 405-406; Physicians Desk Reference 2009, ed 63, pp 1801-1810, 3019-3022; Stoelting:
    Pharmacology and Physiology in Anesthetic Practice, ed 4, pp 117, 378-379, 575).
111
Q

Image on Page 270

  1. Musculocutaneous nerve
  2. Axillary artery
  3. Axillary vein
  4. Ulnar nerve
A
  1. (E)
  2. (B)
  3. (A)
  4. (C)
    Hebl JR, Lennon RL (eds.) Mayo Clinic

Image on Pg 288

112
Q
A. C3-C5
B. T1-T4
C. T5-T12
D. T10-L1
E. S2-S4
  1. Phrenic nerve
  2. Cardiac accelerator fibers
  3. Pudendal nerve
  4. Pain fibers to the uterus
  5. Inhibitory presynaptic fibers to the gastrointestinal tract
A
  1. (A) 903. (B) 904 (E) 905. (D) 906. (C)
    In the normal adult, breathing and coughing can be done exclusively by the diaphragm, which is innervated by
    the phrenic nerve (C3-C5). The heart rate is dependent upon intrinsic pacemaker activity of the SA node, which
    can be affected by the autonomic nervous systems sympathetic nervous systems cardiac accelerator fibers (T1-
    T4) as well as the parasympathetic nervous systems vagus nerve (cranial nerve 10). The first stage of labor pain is
    related to uterine contractions and dilation of the cervix (T10-L1). The second stage of labor is related to both
    uterine pain (T10-L1), as well as birth canal pain which is supplied by the pudendal nerves (S2-S4). The greater
113
Q

Match

A. Glossopharyngeal nerve
B. Internal branch of the superior laryngeal nerve
C. External branch of the superior laryngeal nerve
D. Recurrent laryngeal nerve
E. Cranial nerve XI

  1. Sensory innervation to the posterior one third of the tongue
  2. Motor innervation to the cricothyroid muscle
  3. Sensory innervation below the vocal cords to the carina
  4. Sensory innervation to the mucous membranes of the false cords
  5. Motor innervation to the tonsils
  6. Sensory innervation to the posterior pharynx
  7. Motor innervation to most of the intrinsic muscles of the larynx
  8. Motor innervation to the posterior cricoarytenoid muscle
A
  1. (E) 908. (C) 909. (D) 910. (B) 911. (A) 912. (A) 913. (D) 914. (E)
    When an awake intubation is needed, local anesthetics can be applied topically or by injection. Innervation of
    the airway includes the glossopharyngeal nerve, cranial nerve XI, and branches of the vagus nerve (internal and
    external branches of the superior laryngeal nerve and the recurrent laryngeal nerve). The glossopharyngeal nerve
    provides sensory innervation of the posterior one third of the tongue, the vallecula and the anterior surface of
    the epiglottis (lingual branch), the pharyngeal walls (pharyngeal branch) and the tonsils (tonsillar branch). With
    the exception of the cricothyroid muscle, the recurrent laryngeal nerve of the vagus provides motor innervation
    of all the intrinsic muscles of the larynx. The cricothyroid muscle is supplied by the external branch of the
    superior laryngeal nerve of the vagus. The sensory innervation of the mucosa of the larynx down to the vocal
    folds comes from the internal branch of the superior laryngeal nerve of the vagus, and the sensory innervation
    of the mucosa of the larynx below the vocal folds comes from the recurrent laryngeal nerve of the vagus. The
    muscles of the pharynx are supplied through the pharyngeal plexus from motor fibers from the eleventh cranial
    nerve (accessory nerve) (Barash: Clinical Anesthesia, ed 5, pp 622-623; Brown: Atlas of Regional Anesthesia, ed 3,
    pp 207-222).